You are on page 1of 120

Diagnostic Imaging and Radiology

A 42 year old female presents to her physicians office with complaints of headache and blurred vision. Her symptoms have been occurring over
the last 3 weeks. She has no other complaints. Past medical history reveals that she had a breast cancer removed via lumpectomy 1 year ago, but
did not require chemotherapy or radiation. Family history is positive for polycystic kidney disease in a paternal grandfather and neurofibromatosis in
a maternal grandmother. Physical examination is only remarkable for left peripheral visual field defects. The patient is sent to radiology for a CT
scan of her brain for further evaluation. The study is performed with the administration of intravenous contrast and an image from the scan is shown
here.
Which of the following conditions would you not include in your differential diagnosis for this patient?

1)
2)
3)
4)
5)

Craniopharyngioma
Epidermoid
Meningioma
Berry aneurysm
Metastatic lesion

The correct answer is choice B.

Epidermoid (choice B) is the only condition listed that should not be included in the differential diagnosis for this patient.

The contrast enhanced CT scan of the suprasellar region demonstrates an approximately 2.5 cm extra-axial, densely enhancing, homogeneous,
rounded mass lesion in the left middle cranial fossa just above the medial aspect of the right tentorium, exerting mild mass effect on the inferior
aspect of the left frontal lobe.

Although epidermoid tumors can occur extra-axially in this location, they characteristically are of very low density and do not enhance to any extent
except for perhaps mild ring-enhancement. Hence this choice is an incorrect answer.

The strongest candidates in this differential diagnosis are meningioma (choice C), Berry aneurysm (choice D), and metastatic lesion. Not only is
this lesion radigraphically very similar in appearance to each of these conditions, but the patients history should alert you that she has potential
reason to have each of these conditions. Her history of breast cancer should make you suspicious of a metastatic lesion. Her positive family history
of polycystic kidney disease (PCKD) and neurofibromatosis should alert you to the possibilities of a Berry aneurysm and a meningioma
respectively, as these conditions have high associations as listed.

Meningioma is a benign tumor of the meninges that frequently occurs in the suprasellar region. It is the most common extra-axial tumor in the
cranium and it intensely enhances after the administration of intravenous contrast. There is a known association with neurofibromatosis.
Meningiomas may appear dense on noncontrast CT scans due to the possibility of psammomatous calcifications within. They typically abut a
meningeal surface. They may cause hyperostosis of adjacent bone and only very rarely may cause peritumoral edema. Meningiomas tend to be
easily removed at surgery. The tumor presented in this case is in fact a meningioma.

Berry aneurysm (choice D) is a congenital condition that has a high association with adult PCKD. They are located at the circle of Willis. These
aneurysms are very susceptible to rupture, and lead to the classic history of the worst headache of my life. When they rupture they cause a
subarachnoid hemorrhage. On contrast enhanced CT scan they enhance intensely and are found in the region of the circle of Willis.

Metastatic lesions (choice E) to the suprasellar region tend to be from primary tumors of the lung, breast, kidney, and GI tract. These lesions can be

smooth or irregular in shape, usually enhance homogeneously, and may destroy adjacent bone.

Craniopharyngioma (choice A), a tumor from the remnant of Rathkes pouch more commonly seen in children, is typically a mixed density lesion
that may contain cystic and solid portions, the solid portion of which will enhance variably. They are commonly located in the suprasellar region and
may contain dense calcifications.

A 43 year old male patient comes into the emergency room with complaints of dyspnea, chest pain, fatigue, lower extremity edema, and abdominal
distension and discomfort. He says that these symptoms have all been coming on gradually over the last few years and that he had not sought
medical attention because he did not have health insurance until now. On physical examination the patient appears quite thin, has a blood pressure
of 95 over 55, sinus tachycardia, jugular venous distension, and hepatomegaly with probable ascites. The ER physician sends the patient to
radiology for a chest X-ray which is as shown and you are interpreting it.
Which of the following conditions is a possible etiology of this finding?

1) Tuberculosis
2) Previous radiation therapy to the mediastinum

3) Uremia
4) Rheumatoid disease
5) All of the above

The correct answer is choice E.

The significant finding on the chest X-ray are thickening of the pericardium best seen on the lateral view, small left atrium on the lateral view,
straightening of the left cardiac border, normal sized heart, unremarkable pulmonary vascularity, and a small right pleural effusion. These findings
in combination with the clinical presentation are strongly suggestive of a constrictive pericarditits. Other potential chest X-ray findings in patients
with constrictive pericarditis are pericardial calcification, straightening of the right cardiac border, and enlargement of the SVC, IVC and azygous
vein.

All of the above (choice E) listed diagnoses can cause constrictive pericarditis.

Tuberculous pericarditis (choice A) is not as common an etiology of constrictive pericarditis now as it was in years past when it was the most
frequent cause. In such cases, there would frequently be other signs of tuberculosis infection in the chest such as apical lung disease and
mediastinal lymphadenopathy.

Previous radiation to the mediastinum (choice B) may also produce a constrictive pericarditis, particularly as may be the case with high dose
radiation to the mediastinum used for Hodgkins disease. Uremic patients (choice C) on prolonged hemodialysis are at high risk for constrictive
pericarditis. Constrictive pericarditis in rheumatoid disease (choice D) is relatively rare, but can occur. Clinically evident pericarditis is only present
in about 2% of rheumatoid patients, and only about 10-20% of those would be classified as constrictive.

A 62 year old male presents to his physician with complaints of fatigue, malaise, decreased appetite, weight loss, and vague abdominal pain over
the course of the last 3 months. Past medical history is positive for an episode of hepatitis B 15 years previously. Review of systems reveals that
the patient has been having 3-4 drinks of Scotch whiskey per day for many years. Physical examination is remarkable for slight scleral icterus and

fullness in the right upper quadrant. Laboratory findings confirm a high bilirubin level and show elevated liver function tests as well. The patient is
sent to radiology for an abdominal CT scan, which is performed with intravenous contrast. You are interpreting the study, an image of which is
shown here.
Based on the findings of this CT scan, which of the following conditions would you include in your differential diagnosis for this case?

1)
2)
3)
4)
5)

Hepatocellular carcinoma
Metastatic melanoma
Cavernous hemangioma
Focal nodular hyperplasia
All of the above

The correct answer is choice E.

The CT scan shows a large, round mass in the right lobe of the liver which is enhancing peripherally and toward the center. All of the above choices
(choice E) can manifest with such an appearance on abdominal CT scan and should be included in the differential diagnosis for this CT
appearance. Arterial phase imaging with spiral CT scanning (not shown here) can help to narrow down this differential diagnosis even further as

explained in the following paragraphs.

Hepatocellular carcinoma (choice A) is responsible for 90% of all primary liver malignancies. It tends to present in the 6th to 7th decades of life, and
has a male to female predominance of 2 :1. There is an increased incidence of hepatoma in patients with cirrhosis, hepatitis B or C, and
hemochromatosis. Patients may present with RUQ pain, fever, weight loss, malaise, hepatomegaly, and ascites. Laboratory findings usually
demonstrate elevated alpha-fetoprotien levels and elevated LFTs. Arterial phase enhancement pattern on spiral CT scan frequently shows
abnormal internal vessels or a variegated pattern in hepatocellular carcinoma.

Metastatic melanoma (choice B) and other vascular metastatic lesions such as carcionoid, renal cell carcinoma, thyroid carcinoma can manifest as
a peripherally enhancing intrahepatic mass on CT scan. These lesions frequently demonstrate complete ring enhancement during the arterial
phase on spiral CT scanning.

Cavernous hemangioma (choice C) likewise can manifest as a peripherally enhancing intrahepatic mass on CT scan with progression of
enhancement centrally over time. It is the most common benign liver tumor of adults and affects females more than males. There may be
associated hemangiomas in other organs, and there is an association with Osler-Weber-Rendu disease. These tumors are generally asymptomatic,
but may present with hepatomegaly and abdominal discomfort. Arterial phase enhancement pattern on spiral CT scan frequently shows peripheral
puddling in cavernous hemangiomas.

Focal nodular hyperplasia (choice D) is a manifestion of a reparative process in the liver as a response to injury. This lesion is generally
asymptomatic, but may present with vague abdominal pain, usually in the 3rd to 5th decades of life. It is more common in females than males and
has an increased incidence in women who use oral contraceptives. There may be a central stellate appearing scar see in 15% of cases. Arterial
phase enhancement pattern on spiral CT scan frequently shows a diffuse uniform enhancement pattern in cases of focal nodular hyperplasia.

A 13 year old male presents to his pediatricians office with complaints of pain in his right lower leg, just below the knee. His mother adds that the
boy has also had a low grade fever off and on for awhile and that he seems less active than normal. Physical examination reveals a temperature of
99.8 and moderate tenderness in the region of the proximal diaphysis of the right tibia. The patient is sent to the department of radiology for X-rays
of his right tibia/fibula. An image from that study is shown. Because of the patients age, the 2 most likely bone tumors that you include in your
differential diagnosis are osteosarcoma and Ewings sarcoma.

With regard to osteosarcoma and Ewings sarcoma, which of the following statements is nottrue?

1)
2)
3)
4)
5)

Osteosarcoma is the most common malignant tumor of bone in childhood


Osteosarcoma involves the long bones more frequently than does Ewings sarcoma
Osteosarcoma frequently demonstrates the classic onion-skin appearance of periosteal bone formation
Ewings sarcoma is frequently found in the bones of the pelvis
Ewings sarcoma generally demonstrates more bony lysis than sclerosis as a radiographic finding

The correct answer is choice C.

Ewings sarcoma frequently demonstrates the classic onion-skin appearance of periosteal bone formation, therefore choice C is an incorrect
statement.

Osteosarcoma is a malignant bone tumor that arises from connective tissue and is the most common primary malignancy of bone in childhood
(choice A). It is more common in males than females with a ratio of 3:2, and maximum incidence in the 2nd decade of life. Osteosarcoma involves
the long bones in 80-90% of cases (choice B), frequently in the lower extremities, particularly around the knee joint. It is usually metaphyseal in
location and may extend into the epiphysis. It tends to penetrate the bony cortex and elevate the periosteum, and have a soft tissue mass
associated with it, possibly containing calcifications. Classically seen is a mixture of bone destruction and bone sclerosis on plain films with
spiculation present that radiates perpendicular to the bony cortex. A classic Codmans triangle may also be seen in advanced cases.

Ewings sarcoma is a malignant round cell tumor which is more rare and accounts for approximately 10% of primary malignant bone tumors. It more
often occurs in the 2nd decade of life, but may also present well into the 3rd decade, and has a slight predilection for males over females. It may
occur in the long or flat bones with approximately equal distribution between the two (choice B). When in long bones, it may be located in either the
diaphyseal or metaphyseal region, and when in flat bones, it is frequently found in the pelvis (choice D). Radiologic features include bony lysis with
minimal reactive bone formation and sclerosis. When this tumor reaches the periosteal surface, it stimulates periosteal bone formation which may
be laminated and show the classic onion-skin appearance associated with Ewings sarcoma (choice C). Spiculation may also be present.

A 40 year old woman comes in for her first screening mammogram, which you are now interpreting. You make the following three conclusions on
the mammogram: 8 mm calcified fibroadenoma, scattered vascular calcifications, and a right axillary lymph node that is 9 mm along its longest axis.
What BI-RADS classification should you assign this patient in your impression?
1)
2)
3)
4)
5)

BI-RAD 0
BI-RAD 1
BI-RAD 2
BI-RAD 3
BI-RAD 4

The correct answer is choice C.

BI-RADS is an acronym that stands for Breast Imaging-Reporting and Data System. This system serves to standardize mammography reporting by
putting findings into categories which suggest different guidelines for follow-up. There are seven BI-RADS categories as follows:

BI-RAD 0 - Need additional imaging evaluation - Additional imaging needed before a category can be assigned.

BI-RAD 1 Negative - Continue annual screening mammograms for women over age 40.

BI-RAD 2 - Benign finding - Continue annual screening mammograms for women over age 40.

BI-RAD 3 - Probably benign finding Recommend a 6-month follow-up mammogram for further evaluation.

BI-RAD 4 - Suspicious abnormality present Consider biopsy.

BI-RAD 5 - Highly suggestive of malignancy Biopsy required.

BI-RAD 6 - Known biopsy-proven malignancy - Biopsy has confirmed presence of cancer before treatment has
begun.

The findings of an 8mm calcified fibroadenoma, scattered vascular calcifications, and a right axillary lymph node that is 9 mm along its longest axis
are all benign findings that do not require anymore than routine follow-up. Therefore, this mammogram interpretation is assigned the classification
of BI-RADS 2 (choice C).

A 48 year old heavyset woman comes into the emergency room complaining of severe right upper quadrant (RUQ) postprandial pain. The ER
physician establishes that she has a fever of 102 degrees, a positive Murphys sign, and an elevated white blood cell count of 14,000 with a left
shift. She is sent to the radiology department to have a RUQ ultrasound. The ultrasound technologist asks you to come into the examination room
and look at the real-time ultrasound with her because she is having trouble identifying the gallbladder. You go in and scan the patient and find that
there are very dense linear echoes in the region of the gallbladder fossa with limited through transmission.
Which of the following diagnoses do you suspect at this point?
1)
2)
3)
4)
5)

Acalculous cholecystitis
Chronic cholecystitis
Gallstone ileus
Emphysematous cholecystitis
Porcelain gallbladder

The correct answer is choice D.

The very dense linear echoes in the region of the gallbladder fossa with limited through transmission in a patient with signs and symptoms of acute
cholecystitis should lead you to suspect that the patient may have emphysematous cholecystitis (choice D). Air present in the gallbladder wall will
not transmit ultrasound waves and will read as very dense linear echoes with limited through transmission.

Emphysematous cholecystitis may also be referred to as gangrenous cholecystitis. It is usually associated with gas-forming organisms such as
clostridium perfringens, clostridium Welchii, and E. coli. The mortality rate for this condition is approximately 15%.

Porcelain gallbladder (choice E) due to calcium incrustation of the gallbladder wall can also cause dense linear echoes with limited through

transmission when scanning the gallbladder wall, but, patients with porcelain gallbladder are usually asymptomatic and would not present with
symptoms and signs of acute cholecystitis.

Gallstone ileus (choice C) is due to the erosion of a gallstone through the gallbladder wall into the GI tract subsequently causing an obstruction,
frequently at the level of the ileo-cecal valve. These patients may have gas in the biliary tree and gallbladder, but would not have gas within the
gallbladder wall.

Acalculous cholecystitis (choice A) and chronic cholecystitis (choice B) would not have the show dense linear echoes with limited through
transmission when scanning the gallbladder wall, rather, visualization of the gallbladder with ultrasound would be possible.

You are the radiology resident assigned to the high magnetic field MRI scanner for the day. You begin your day by reviewing all the charts of
scheduled patients for the day, reviewing patient history and reason for performing the MRI scan so that you can tailor each examination
appropriately. In your review of the charts, you find the following information on four of the scheduled patients.
In which of these four patients would you not proceed with the MRI scan?
1) A 38 year old male that weighs 400 lbs
2) A 28 year old woman who is 6 months pregnant that will not require the administration of IV contrast material
3) A 52 year old male that had a left total hip replacement in 2009
4) A 55 year old female with dental implants
5) A 75 year old male with an insulin pump

The correct answer is choice E.

An insulin pump (choice E) is a contraindication to performing an MRI because it contains magnetic components.

Other absolute contraindications to MRI are magnetically activated cardiac pacemakers, defibrillators, neurostimulators, cochlear implants, as well
as metallic fragments within the eye, ferromagnetic intracranial aneurysm clips, nongraphite spinal cord tongs, and Swan-Ganz catheters. Patients
weighing over 425 lbs exceed the weight limit of most MRI tables posing a risk to the patient and MRI equipment; scanning in such instances

should be avoided.

Relative contraindications to MRI include implanted lead wires, prosthetic heart valves (unless dehiscence is suspected, then it would be
contraindicated), body hemostatic clips, tattoos and certain makeup, first trimester of pregnancy as long term effects on the fetus are not well
known.

Choice A a 400 lb patient could proceed to have an MRI as this weight is below the maximum weight limit for the MRI table. Choice B a patient
who is in her 6th month of pregnancy could proceed with an MRI as the concern in pregnancy for safety of the fetus is only in the first trimester, but
intravenous contrast during MRI is generally avoided for the entire pregancy as its long-term effects on the fetus are not well known. Choices C and
D dental implants and joint prostheses are not contraindications to MRI scanning, although they may cause artifacts on images obtained.

A 22 month old boy is brought to the emergency room by his father who states that the boy suddenly developed difficulty breathing, fever, and is
making a funny sound when he breathes in. He also states that the boy was found earlier in the day having gotten into a bowl of peanuts
inadvertently left on the coffee table and he wonders if the boy might have aspirated a peanut. Physical examination reveals that the boy is holding
his head forward, is in obvious respiratory distress, and has inspiratory stridor. He also has a temperature of 102. The patient is immediately sent to
radiology for AP and lateral films of his neck, the lateral of which is shown here.
What is your diagnosis in this patient?

1)
2)
3)
4)
5)

Croup
Peanut lodged in upper trachea
Bronchiolitis
Epiglottitis
Retropharyngeal abscess

The correct answer is choice D.

The lateral X-ray of the neck shows a markedly enlarged epiglottis and aryepiglottic folds, consistent with the diagnosis of epiglottitis (choice D).

Epiglottitis is a life-threatening infection due to Hemophilus influenzae (sometimes pneumococcus or streptococcus group A) which causes
significant edema of the epiglottis and aryepiglottic folds. The most common ages of onset are 3-6 years of age. Symptoms are abrupt onset of
respiratory distress with inspiratory stridor and dysphagia. Diagnosis is readily made on a lateral neck X-ray which will show enlargement of the
epiglottis and aryepiglottic folds. The epiglottis can be readily identified as it is projected atop the hyoid bone on the lateral view, and the
aryepiglottic fold extending posterior and inferior to it. Note should be made that in suspected cases of epiglottitis, the lateral neck image should be
obtained with the patient in the upright position, as supine positioning may hasten obstruction of the airway. The life-threatening danger of this
condition is due to the risk of closure of the airway by the enlarged, edematous epiglottis and aryepiglottic folds.

Croup (choice A) is an acute laryngotracheobronchitis of viral origin. In this condition, the critical area of airway narrowing is in the subglottic region.
Peak incidence of this condition is 6 months 2 years of age. Croup is usually associated with a few day history of respiratory tract infectious
symptoms and a brassy cough. AP neck films show the classic steeple sign which represents the subglottic edema and results in a steeple-like
appearance of the subglottic airway. An example of a PA radiograph of croup with the classic steeple sign is shown here:

It is important to make the distinction between croup and epiglottitis, and in croup, the epiglottis is not enlarged, therefore this choice is incorrect.

A peanut lodged in the upper trachea (choice B) may show as a soft tissue filling defect in the tracheal airway, or may not show up on a plain X-ray
at all. Although the possibility of peanut aspiration was raised in this patient, and symptoms of respiratory distress and stridor may occur, there
would generally not be the presence of fever and the epiglottis would definitely not be enlarged on lateral neck X-ray, hence this choice is incorrect.

Bronhciolitis (choice C) is an incorrect choice as it is a viral inflammation (commonly due to respiratory syncytial virus RSV) of the lower airways
in infants less than 1 year of age which causes air-trapping in the lungs. Radiographic findings include hyperinflation on chest X-ray and increase in
bronchiolar markings.

Retropharyngeal abscess (choice E) may occur in infants secondary to a nasopharyngeal infection, or secondary to a penetrating injury, such as
one incurred by falling on a Popsicle stick which pierces the retropharyngeal mucosa. Radiographic signs include thickening of the retropharyngeal
space seen on lateral neck film with anterior displacement of the upper airway. These signs are not seen in this case and therefore this choice is
incorrect.

A 3 year old girl is brought to her pediatricians office by her mother for a routine visit. On physical examination, he patients blood pressure is
elevated at 140/90 and the pediatrician detects a mass palpable in the patients right upper quadrant. Upon further questioning of the patients
mother, it is revealed that the child has intermittently complained of stomach aches. The patient is sent to radiology for evaluation of the
abdominal mass by ultrasound, to be followed by abdominal CT scan as necessary. You are interpreting the abdominal CT scan which has been
performed with the administration of intravenous contrast. An image from that CT scan is shown.
Of the following choices, which is the most likely diagnosis in this patient?

1)
2)
3)
4)
5)

Right-sided hydrohephrosis
Neuroblastoma
Wilms tumor
Renal cell carcinoma
Angiomyolipoma

The correct answer is choice C.

The abdominal CT scan shows an extremely large, heterogeneously enhancing mass of the right kidney. The determination that the mass is
intrarenal is made by the visible splaying of the nephrogram by the mass posteriorly. This is an important differential diagnostic point in this case.

Wilms tumor (choice C) is the most common primary malignant renal tumor of childhood. It occurs in approximately 1:10,000 individuals, with a
peak age of presentation at 3-4 years old. Discovery of Wilms tumor is most often made after detection of a palpable mass on physical
examination, or possibly after coincidental trauma and imaging. Hypertension may be present in up to 50% of cases. Wilms tumor arises
embryologically from cells of the metanephros. Metastases may occur to the lungs, liver, and regional lymph nodes. Occasionally, it may extend
into the renal vein and IVC. It can also be bilateral in a small percentage of cases. CT generally demonstrates a large, round, heterogeneous,
intrarenal mass with peripheral enhancement and a lower density center due to necrosis. There may be a thick pseudocapsule present and
calcifications within the mass. Treatment is via nephrectomy and some potential combination of chemotherapy and radiation. Cure rates are
reported as high as 90%.

Right-sided hydronephrosis (choice A) is not an appropriate choice in this case as it would not have the appearance of a solid intrarenal mass.

Neuroblastoma (choice B), although another common abdominal mass in children, is an extrarenal mass, generally originating in the adrenal gland,
and would not demonstrate the intrarenal appearance of the mass as in this case, and is therefore incorrect.

Renal cell carcinoma (choice D), although may have a similar CT appearance, is primarily a disease of adulthood, and exceedingly rare in children,

therefore an incorrect choice.

Angiomyolipoma (choice E) is a benign tumor of adulthood, which is also extrarenal, frequently originating in the adrenal gland, and therefore
incorrect as well.

You are the radiology resident assigned to the CT scanner for the day. You begin your day by reviewing all the charts of scheduled patients for the
day, reviewing patient history and reason for performing the CT scan so that you can tailor each scan appropriately. In your review of the charts,
you determine that the following four patients would all require the administration of IV contrast material during the CT scan.
In which of these four patients would you proceed with the CT scan?
1)
2)
3)
4)
5)

43 year old male that weighs more than 425 lbs


32 year old woman that is pregnant
72 year old male with chronic renal failure on dialysis
49 year old woman with a past history of an IV contrast reaction who has not been pretreated for the examination
None of above; each patient listed has a contraindication to having a CT scan performed

The correct answer is choice C.

A patient who has chronic renal failure and is on dialysis (choice C) may receive IV contrast and have a CT scan performed as the IV contrast will
be cleared during the patient's dialysis.

Pregnancy (choice B) is an absolute contraindication to CT scan as the modality employs radiation which can pose a significant risk to the fetus for
developing congenital anomalies.

Other relative contraindications include patient weight greater than 425 lbs (choice A) as such weight exceeds that which can be safely supported
by the CT table and therefore would pose a risk to the patient as well as the CT equipment, and a patient history of contrast reaction unless the
patient has been pretreated with corticosteroids (choice D). Other relative contraindications include renal insufficiency and hyperthyroidism/goiter if

intravenous contrast is to be administered during the CT scan.

Choice E none of the above, is incorrect as choice C is a correct answer.

A one day old male newborn in the hospital nursery is irritable and has vomited most of his first feeding. After the newborns second feeding, he
again is irritable and vomits. The pediatrician covering orders an abdominal X-ray which you are now interpreting. The film shows a classic double
bubble sign.
Which of the following conditions would you not include in your differential diagnosis for this case?
1)
2)
3)
4)
5)

Annular pancreas
Duodenal atresia
Midgut volvulus with axis at ligament of Treitz
Hypertrophic pyloric stenosis
Preduodenal positioning of portal vein

The correct answer is choice D.

Hypertrophic pyloric stenosis may also cause GI obstruction in the newborn, but the classic finding on X-ray in this condition is the single bubble
sign, as a result of an enlarged gas containing stomach and the inability of the gas to pass through the pylorus into the duodenum.

The double bubble sign is indicative of duodenal obstruction, either intrinsic or extrinsic. The two bubbles represent a distended gas-filled stomach
and duodenum, with lack of gas distal to the duodenum.

Examples of intrinsic obstruction of the duodenum are: duodenal atresia (choice B), duodenal stenosis, and duodenal webs. Examples of extrinsic
duodenal obstruction are: annular pancreas (choice A), midgut volvulus with axis at ligament of Treitz (choice C), and preduodenal positioning of

the portal vein (choice E).

The double bubble sign may also be seen on ultrasound of the abdomen in these conditions.

A 28 year old woman comes into the emergency room with lower abdominal pain and vaginal bleeding, with the pain being significantly worse than
that with her normal menses. Upon questioning by the ER resident, the patient reveals that her last period was actually 2 months prior. Physical
and pelvic examinations reveal a slightly enlarged uterus with pain elicited upon manipulation of the cervix. There is a slight suggestion of a left
adnexal mass. Urine pregnancy test is positive, serum beta-hCG is 2,000 and transabdominal ultrasound does not reveal an intrauterine
gestational sac. The ER resident suspects that the patient has an ectopic pregnancy and you, the radiology resident, are called to the ER to
perform a transvaginal ultrasound for further evaluation. An image from that study is shown.
Of the following transvaginal ultrasound findings that may be seen in ectopic pregnancy, which finding is the most diagnostic?

1)
2)
3)
4)
5)

Fluid in the pouch of Douglas


Ectopic mass
Ectopic mass and fluid in the pouch of Douglas
Absence of intrauterine pregnancy
Ectopic cardiac activity

The correct answer is choice E.

The most specific findings for ectopic pregnancy on transvaginal ultrasound in order of specificity are ectopic cardiac activity (choice E), followed by
ectopic gestational sac, ectopic mass and fluid in the pouch of Douglas (choice C), fluid in the pouch of Douglas (choice A), ectopic mass (choice

B), and absence of intrauterine pregnancy (choice D).

Ectopic pregnancy occurs in up to 1.9% of pregnancies and ruptured ectopic pregnancies are a life threatening condition, the incidence of which
has diminished with the advent of transvaginal ultrasound leading to more rapid diagnosis and treatment.

A 29 year old healthy male athlete who has sustained an injury to his left knee is sent by his orthopedist to the radiology department for an MRI of
the left knee which you are interpreting. You determine that the knee ligaments and cartilages are normal, and that there is no evidence of
traumatic injury to the left knee except for the presence of a small joint effusion. You do however make the finding of a very well defined 3 cm
eccentric lesion of the tibial diaphysis that is low intensity on T1 weighted images and low intensity on T2 weighted images. Plain X-rays of the
patients left knee are unavailable for viewing at this time.
Of the following benign lesions of bone, which one would demonstrate low signal intensity on T1 weighted images and low signal intensity on T2
weighted images?
1)
2)
3)
4)
5)

Simple bone cyst


Benign osteoblastoma
Enchondroma
Chondromyxoid fibroma
Brodies abscess

The correct answer is choice B.

During an MRI scan, the patient is placed into a very high magnetic field that causes randomly aligned hydrogen atoms to align parallel with the
newly applied magnetic field. Radiofrequency pulses are then applied which elevate the hydrogen atoms to a higher level energy state (repetition
time = TR = time between pulses applied). When the radiofrequency pulses are terminated, the hydrogen atoms release energy as they return to
their pre-excited state. This energy released by hydrogen atoms is measured in MRI scanning as the relaxation time, or TE. Suffice it to say that
when a long TR is applied and the long TE of hydrogen atoms is measured, signal is manifested as high intensity on T2 weighted images.

Benign osteoblastoma (choice B) is a benign lesion of bone composed of connective tissue that tends to produce osteoid and bone in its matrix. It

is more common in males than females and tends to occur in the 2nd and 3rd decades of life. Mild pain is generally the presenting symptom with
some tenderness possible at the site. This lesion most frequently occurs in the long tubular bones and in the posterior elements of the vertebrae
and characteristically has low signal intensity on T2 weighted images due to the presence of osteoid and bone within the lesion, and a low
hydrogen content.

The remaining choices in the benign lesions of bone listed all have a high hydrogen content and hence high signal intensity on T2 weighted
sequences simple bone cyst (choice A), enchondroma (choice C), chondromyxoid fibroma (choice D), and Brodies abscess (choice E) would all
demonstrate low signal intensity on T1 weighted images and high intensity signal on T2 weighted images due to content within the lesions as
follows: simple bone cyst fluid content, enchondroma - high water and mucopolysaccharide content, chondromyxoid fibroma chondroid and
myxoid content, Brodies abscess inflammatory content.

A 52 year old male presents to the emergency room in winter with a one week history of flu-like symptoms which have now suddenly gotten much
worse with a fever of 102 degrees, and onset of a very productive cough. You are reading his chest X-ray (shown here) in which you suggest that
he has pneumonia.
Which of the following portions of the lung(s) show air-space consolidation?

1)
2)
3)
4)
5)

Right middle lobe (RML)


Right lower lobe (RLL)
Left lower lobe (LLL)
RLL & LLL
RML, RLL & LLL

The correct answer is choice E.

The RML, RLL, and LLL (choice E) all show air-space consolidation as evidenced by silhouetting of the right heart border, the right hemidiaphragm,
and the left hemidiaphragm respectively.

The history and CXR appearance in this patient favor a diagnosis of influenza with the development of a secondary bacterial pneumonia.

None of the other choices A D are all inclusive of the RML, RLL and LLL, and are therefore incorrect.

A 42 year old woman is sent from her gastroenterologists office to radiology for a CT scan of her abdomen. She has been having abdominal pain,
bloody diarrhea, and weight loss. A mid abdominal CT image is as shown.
This radiographic appearance is consistent with which of the following conditions?

1)
2)
3)
4)
5)

Ischemic colitis
Ulcerative colitis
Pseudomembranous colitis
Lymphoma
All of the above

The correct answer is choice E.

The mid abdominal CT image shows the transverse colon with a diffusely thickened wall and the classic sign of thumbprinting. The differential
diagnosis of thumbprinting of the colon includes ischemic colitis (choice A), ulcerative colitis (choice B), pseudomembranous colitis (choice C),
and lymphoma (choice D).

Ischemic colitis (choice A) would likely show segmental disease due to occlusion of a particular vessel. Ulcerative colitis (choice B) would tend to
involve the rectum, which would help to distinguish it from ischemic colitis and Crohns colitis. Pseudomembranous colitis (choice C) due to
clostridium difficile usually occurs after a course of antibiotic therapy and tends to involve the entire colon as a potentially distinguishing feature,
although ulcerative colitis can do so as well. Thumbprinting with lymphoma (choice D) is often asymmetric and irregular, and its onset would be
insidious as compared to the other conditions listed.

Other conditions that should also be included in the differential for thumbprinting of the colon are endometriosis in women, amyloidosis, hereditary
angioneurotic edema, Crohns disease (although thumbprinting is less commonly seen than with ulcerative colitis) and pneumatosis intestinalis (in
which case air would be identified in the colonic wall).

You are about to perform an abdominal CT scan on a 67 year old male who has been diagnosed with colon cancer.
Regarding the radiation received from this CT scan, what is the approximate time period for equivalent effective dose from natural background
radiation?
1)
2)
3)
4)
5)

2.5 days
26 weeks
1 year
3 years
9 years

The correct answer is choice D.

An abdominal CT scan has an approximate time period for equivalent effective dose from natural background radiation of 3 years (choice D).

In general, the higher the dose and duration of radiation during the study, the greater the time period for equivalent effective dose from natural
background radiation.

2.5 days (choice A) is the approximate time period for equivalent effective dose from natural background radiation for a typical chest X-ray. 26
weeks (choice B) is the approximate time period for equivalent effective dose from natural background radiation for a typical lumbar spine X-ray. 1
year (choice C) is the approximate time period for equivalent effective dose from natural background radiation for a typical intravenous pyeologram
(IVP). 9 years (choice E) would be the approximate time period for equivalent effective dose from natural background radiation for three abdominal
CT scans.

A 73 year old man with hypertension secondary to chronic renal failure presents to the emergency room after experiencing tearing chest pain which
radiates to his back. He is fearful that he is having a heart attack. Physical examination reveals a bruit over the patients upper back and
asymmetrical pulses in the lower extremities. Blood pressure is 160/95. Due to the patients chronic renal failure, an MRI is elected as the imaging
modality to make his diagnosis, an oblique coronal image of which is shown here.
With regard to this patients diagnosis, which of the following radiographic studies readily makes this diagnosis?

1)
2)
3)
4)
5)

CT scan
MRI
Arteriography
Trans-esophageal ultrasound
All of the above

The correct answer is choice E.

All of the above (choice E) answers are correct in this case as each of the imaging modalities (CT scan A, MRI B, arteriography C, and transesophageal ultrasound D) can make this patients diagnosis dissecting aortic aneurysm type III.

The wall of the aorta, from lumen to exterior, is comprised of intima, media, and adventitia. A dissecting aortic aneurysm results from a tear of the
intima, leading to the formation of a false lumen in the media portion of the aortic wall. Blood may flow in this false lumen and rejoin the true lumen
more distally, or blood may simply clot within the false lumen and remain static. Dissecting aortic aneurysms are classified into three types:

Type I dissection involves the ascending and descending aorta, originating just above the aortic valve.

Type II dissection involves only the ascending aorta, originating just above the aortic valve.

Type III dissection involves only the descending aorta, originating below the level of the left subclavian artery.

This diagram illustrates the three classifications of aortic dissection, but uses A, B, C instead of I, II, III:

There is a male to female predominance of this condition of 3:1 and risk factors include hypertension, Marfans syndrome, coarctation of the aorta,
valvular aortic stenosis with post-stenotic dilatation, and presence of a prosthetic aortic valve. Symptoms and signs may include sharp, tearing
chest pain that radiates to the back, bruit, aortic regurgitation, asymmetric peripheral pulses, bowel ischemia, neurological deficits, cardiac
ischemia, and oliguria. These findings all depend on which vessels become involved in the dissection. Trans-esophageal ultrasound (choice D) can
be used to make the diagnosis of dissecting aortic aneurysm, and due to the proximity of the descending aorta to the esophagus, this mode is
particularly useful in diagnosing Type III aortic dissections. Conventional arteriography (choice C) is a very effective method of diagnosing a
dissecting aortic aneurysm, but this method is rarely used currently as it has been supplanted with CT scanning, and to a lesser extent, MRI. The

classic findings on arteriography are the demonstration of an intimal flap (the longitudinal physical structure that separates the true from the false
lumen) and the presence of a false lumen. Liekwise, CT scanning (choice A) and MRI (choice B) readily show the intimal flap and false lumen, and
are much less invasive and more readily available examinations, especially CT scanning.

A 48 year old male presents to his physician with complaints of dysphagia for solids over the last 6 weeks with mild retrosternal pain while eating.
Review of systems reveals that the patient smoked 1 pack of cigarettes per day from the ages of 20-32 years. He consumes 5-6 drinks of alcohol
per week. Physical examination and screening laboratory tests are entirely normal. The physician sends the patient to radiology for an upper GI
(UGI) examination for further evaluation. You are performing the UGI and note an abnormality of lower esophagus during fluoroscopy. A spot film of
the lower esophagus is shown here.
The most likely diagnosis in this patient is:

1)
2)
3)
4)
5)

Achalasia
Hiatal hernia
Esophageal leiomyoma
Esophageal cancer
Esophageal varices

The correct answer is choice C.

The spot film of the lower esophagus shows a large (approximately 10 cm), round, smooth intramural filling defect of the distal left lateral
esophageal wall just above the gastro-esophageal junction. Note is made that the esophageal mucosa is intact, hence smooth. These findings are
most consistent with an esophageal leiomyoma (choice C).

Esophageal leiomyoma is the most common benign tumor of the esophagus. Most of these tumors are asymptomatic unless large enough to cause
dysphagia and substernal pain. Leiomyomas are most frequently found in the distal third of the esophagus. These tumors are determined to be
intramural on barium studies because of their nonacute angular interface with adjacent unaffected esophagus. (Mucosal lesions tend to have an
acute angular interface with adjacent normal mucosa.) These lesions may rarely contain enough calcification to be detected on X-ray, and if so, is
diagnostic as no other lesions of the esophagus are known to contain calcifications. Leiomyomas of the esophagus rarely ulcerate, unlike their
counterparts in the stomach. Few esophageal leiomyomas will undergo malignant transformation into leiomyosarcomas.

Achalasia (choice A) is a functional obstruction of the distal esophagus due to incomplete relaxation of the lower esophageal sphincter combined
with decreased motility of the esophagus. This condition is due to a paucity of myenteric plexes and cholinergic innervation of the distal esophagus.
Dysphagia is a symptom of achalasia, but radiologic findings on barium esophagram demonstrate a gradual, smooth narrowing of the distal
esophagus with intact mucosa offering a classic beak-like appearance of the distal barium column. Achalasia is therefore an incorrect answer and
an image of this condition is shown here:

Hiatal (hernia choice B) is an extremely common problem that is defined as a portion of the stomach lying above the diaphragm within the thoracic
cavity. Symptoms may include heartburn, pain, and dysphagia. Diagnosis is made by determination of the gastroesophageal junction (Shatzkis
ring) or gastric mucosa above the diaphragm within the thoracic cavity. Therefore, hiatal hernia is an incorrect choice in this case.

Esophageal cancer (choice D) is an incorrect choice in this case as radiographically on barium study it would appear as an irregular, polypoid,
fungating mass with interruption of the mucosa, and possible deep ulceration. In addition, the mass would have acute, overhanging margins with
adjacent normal mucosa.

Esophageal varices (choice E) is also an incorrect choice in this case as they appear as smooth, serpiginous filling defects of the distal esophagus
that run parallel with the esophagus. Overlying mucosa is also normal in this condition and changes in form may be noted with respiration on
fluoroscopy.

A 74 year old man is seen in GI clinic with complaints of decreased appetite, early satiety, and weight loss. The patients physical examination is
unremarkable but Hemoccult fecal blood test is positive. The patient is sent to radiology by the gastroenterologist for an upper GI examination,
which you perform. An image from that examination in which you suspect that the patient has a malignant ulcer is as attached.
Regarding malignant vs. benign ulcers in the stomach, all of the following statements about malignant ulcers are true except for:

1)
2)
3)
4)
5)

Ulcer location is within the margin of the gastric lumen


Ulcer location is more likely to be on the greater curvature than the lesser curvature
Gastric folds radiate toward the ulcer
Ulcer is irregular in shape with a nodular floor
Presence of Carman meniscus sign

The correct answer is choice C.

Gastric folds in a malignant ulcer typically do not radiate toward the ulcer crater (choice C). This finding is a characteristic of benign ulcers. A

benign gastric ulcer with folds radiating toward the crater and the crater projecting outside the gastric lumen is shown here:

Folds in malignant ulcers tend to be irregular, not directed toward the ulcer, and may be clubbed or amputated in the region of the ulcer.

Malignant ulcers tend to project within the margin of the gastric lumen (choice A) where as benign ulcers tend to project outside the margin of the

gastric lumen.

Malignant ulcers tend to favor the greater curvature (choice B) as opposed to benign ulcers which favor the lesser curvature. (There is an obvious
exception to this general characteristic in the case presented here.)

Malignant ulcers tend to be irregular in shape with a nodular floor (choice D) whereas benign ulcers tend to be quite regular with sharp margins.

The Carman meniscus sign is a radiographic appearance of gastric malignancy that is created by a large, flat ulcer with heaped-up edges which
trap a lenticular barium collection that is convex toward the lumen when the edges are folded upon themselves during compression.

A 53 year old woman presents to her family physician and complains of progressive hearing loss and tinnitus in her right ear. She also complains of
occasional dizziness. Her symptoms have been progressively getting worse over the last 6 weeks. Neurological examination suggests a
sensorineural hearing loss on the right, but no other significant abnormality. The patient is sent to radiology for an MRI of her brain. A T1 weighted
image post intravenous contrast from that study is shown here.
Which of the following is the most likely diagnosis in this patient?

1)
2)
3)
4)
5)

Meningioma
Acoustic schwannoma
Epidermoid tumor
Arachnoid cyst
Metastatic lesion

The correct answer is choice B.

The MRI shows an intensely enhancing lesion of the right internal auditory canal that spills out into the region of the cerebellopontine angle in the
shape of an ice-cream cone, a very typical appearance for an acoustic schwannoma (choice B).

Acoustic schwannoma is the most common lesion of the cerebellopontine angle (CPA), making up 70-80% of tumors in this location. They occur in
females more commonly than males at a ratio of 2:1, and about 5% are associated with neurofibromatosis type II. Acoustic schwannomas typically
present anywhere from 35 60 years of age with sensorineural hearing loss on the side of the tumor, associated tinnitus, and possible dizziness
and vertigo. Radiologic findings on MRI, which is the imaging study of choice, typically are seen as mass lesions aligning themselves along the
course of cranial nerve VII, perhaps enlarging the internal auditory canal (IAC), extending out of the IAC, and forming an ice-cream cone shaped
mass which intensely enhances in a homogeneous manner post intravenous administration of gadolinium.

Meningioma (choice A) is the second most common CPA tumor, accounting for approximately 10-15% of tumors in this location. Meningiomas in
the CPA characteristically have a broad base on the side of their meningeal origin and project a more spherical margin toward the opposite side,
and for this reason, choice A is incorrent. Meningiomas also tend to enhance intensely and homogeneosly. If these CPA tumors become large
enough, they may also produce sensorineural hearing loss. As a distinction from acoustic schwannomas, if they do extend into the IAC, they not
typically enlarge it.

Both epidermoid tumors (choice C) and arachnoid cysts (choice D), although cause can cause mass lesions of the CPA, they do not enhance post
intravenous gadolinium administration, and are therefore incorrect choices in this case.

Metastatic lesions can occur at the CPA (choice E) and cause enhancing lesions, but these lesions are far less common than acoustic
schwannomas and tend to be multiple when they do occur. This choice is therefore incorrect.

An 84 year old man who has had multiple recurrent episodes of suspected aspiration pneumonia over the last 2 years is being evaluated in the
radiology department with an upper GI (UGI) examination to assess for gastroesophageal reflux (GER) as a potential cause for the pneumonias.
You are the radiologist performing the examination and you are able to demonstrate GER fluoroscopically during the examination. The final
overhead image of that study is presented here which shows evidence of gastroesophageal reflux and scarring in the left mid lung secondary to the

patients history of recurrent episodes of aspiration pneumonia. Dense calcification of costrochondral cartilage (particularly on the right) as well as
of the iliac arteries bilaterally are also noted.
With relationship to radiopacity seen on plain x-ray images, which of the following is correct in listing these opacities from the most radiopaque to
the least?

1)
2)
3)
4)
5)

Bone, metal, soft tissue, fat, air


Metal, bone, soft tissue, air, fat
Metal, bone, soft tissue, fat, air
Metal, bone, fat, soft tissue, air
Bone, metal, fat, soft tissue, air

The correct answer is choice C.

There are 5 basic radiodensities encountered in plain x-ray which are listed here from highest to lowest in their radiopacity: metal, bone, soft tissue,
fat, air (choice C). The remainder of the choices (A, B, D, & E) contain the wrong order of these components radiodensity and are therefore all
incorrect answers.

The image seen on plain x-rays is the result of the degree of penetration of x-rays through the body, or body parts, that ultimately reach and expose
the x-ray cassette detectors. The more radiopaque a material is, the less penetration of the x-ray, and the less exposed the cassette detectors
become. Conversely, the less radiopaque a material is, the more penetration of the x-ray, and the more exposed the cassette detectors become.
Said another way, the degree of radiodensity of a material is inversely proportional to the degree of exposure on x-ray cassette detectors through
that material. No exposure results in pure white on x-ray image, complete exposure results in pure black on x-ray image. Radiodensities recorded
by x-ray cassette detectors in imaging of the body lie in between pure white and pure black on x-ray image, and it is the relative relationships of
those radiodensities that allow the interpreter of the x-ray image to extract information from the image.

There are 5 basic radiodensities encountered in plain x-ray listed here from highest to lowest radiopacity: metal, bone, soft tissue, fat, air. (The soft
tissue category contains water-based bodily fluids; fat-based bodily fluids such as chyle would be contained in the fat density category.) The
radiodensities of the metal and bone categories have to do with the atomic numbers of the components within those categories. The higher the
components atomic number is, the more absorption of the x-ray beam, the more radiopaque, the less exposure of the x-ray cassette, and the
whiter the image. So, with barium, which is a metal, having an atomic number of 82 and calcium within bone having an atomic number of 20,
barium will appear more radiopaque than bone. (Bone is further less radiopaque than barium due to the presence of organic matrix within bone
contributing to further radiolucency than that of pure calcium.) The relative radiodensity of the remaining categories of soft tissue air and fat, is the
result of the physical densities of their compositions.

Relative to the overhead x-ray image shown, barium, a metal is seen within the GI tract from the esophagus to the colon, and appears as the most
radiopaque material on the image. The next most radiopaque component is calcium seen in bone, vessels and chostrochondral cartilage. Following

in radiopacity is soft tissue density, well demonstrated here by the liver. Next is fat density, well seen on this film in the subcutaneous region on the
right, and lateral to the psoas muscles bilaterally. Finally, the least radiopaque component seen is that of air in the lungs.

Also relative to this discussion is the fact that the thicker a specific tissue component is, the more absorption of the x-ray beam, the less exposure
of the x-ray cassette, and the whiter the resulting image. This principle is exemplified by the comparison of a chest or abdominal image taken of a
slightly built patient vs that of a heavy-set patient using the same kilovoltage and time of exposure. The heavy-set patients image will appear lighter
than that of the slightly built patient due to the increased absorption of the x-ray beam by the greater thickness of tissue present in the more heavyset patient.

A 72 year old male presents to a walk-in clinic with complaints of a persistent dry cough, dyspnea, right-sided chest wall pain, fatigue, fever, night
sweats and weight loss, all having progressed over the last year. When asked why the patient had not sought medical attention sooner, he
responded that he did not have health insurance and could not afford it. Past medical history revealed that he worked in a shipyard from the ages of
20-40. On physical examination the patient was cachectic with a temperature of 100. Breath sounds were markedly diminished on the right side,
and completely absent at the right base. A chest X-ray was obtained at the walk-in clinic, which prompted the physician to order a CT scan of the
patients chest, which you are interpreting. A representative image of the examination is presented. No parenchymal lung disease is noted on other
images of the CT scan.
What is your primary working diagnosis in this patient?

1)
2)
3)
4)
5)

Lymphoma
Tuberculous empyema
Pancoast tumor
Malignant mesothelioma
Metastatic disease

The correct answer is choice D.

This patients chest CT scan shows extensive, bulky, lobular masses of the pleura on the right side. The distal esophagus and IVC appear to be
encased as well. There is a large right-sided pleural effusion and a hint of ascites under the right diaphragmatic crus. In a patient with an
occupational history of working in a shipyard where asbestos exposure is highly likely, the primary working diagnosis in this patient is malignant
mesothelioma (choice D).

Malignant mesothelioma is an uncommon tumor of mesothelial cells that line the pleura, usually associated with a history of chronic asbestos

exposure, although asbestos exposure is not a prerequisite for the disease. The incidence of this disease is increasing, and currently occurs in the
United States at a rate of about 2,500 new cases per year. Radiographic findings include a large pleural effusion in approximately 90% of cases
which may progress to a fibrothorax. This may be the only detectable finding on plain chest X-ray. Pleural masses and/or pleural rind are usually
better visualized on chest CT scan. Chest wall invasion is not uncommon in later-stage cases. Definitive diagnosis is made by biopsy. The earlier
the diagnosis is made, the better the outcome for the patient.

Lymphomatous involvement of the pleura (choice A) can occur in association with mediastinal and parenchymal involvement in the chest, but with
this patients history of occupational exposure, it would not be the primary working diagnosis.

Tuberculous empyema (choice B) is most frequently associated with a calcified pleural rind and evidence of parenchymal disease of the upper
lobe(s).

Pancoast tumor (choice C) refers to a tumor that involves the apical pleura and would not be present at the lung base.

Metastatic disease to the pleura (choice E) is uncommon but can occur with the most likely primary tumors being bronchus, breast, ovary and GI
tract. This condition is also frequently associated with a pleural effusion. Although this diagnosis should be included in the differential, malignant
mesothelioma is more likely in this patient due to his occupational exposure.

A 57 year old woman who is a long-time cigarette smoker has recently been diagnosed with biopsy proven lung carcinoma. She is in the radiology
department today undergoing an abdominal CT scan as part of her work-up to rule out metastatic disease. While reviewing her abdominal CT scan,
you are going to be particularly interested in her adrenal glands, liver, and look for potential lymphadenopathy as well as any other abnormality
within the abdomen. While observing the initial noncontast CT scan being done, you notice that there is a well circumscribed 3 cm round lesion of
the left adrenal gland with a low attenuation center (see associated image presented). Both size and density of the lesion can be helpful in
suggesting whether the lesion may be benign or malignant so you ask the CT technologist to go back after the scan to do ultra-thin sections
through the left adrenal gland and obtain Hounsfield measurements of the visualized lesion before the administration of IV contrast. The Hounsfield
units obtained at that point are in the 8-10 range.
Regarding lesions of the adrenal glands, which of the following could possibly demonstrate Hounsfield density unit measurements in the 8-10

range?

1)
2)
3)
4)
5)

Myelolipoma
Pheochromocytoma
Myelolipoma and adrenal adenoma
Myelolipoma and pheochromocytoma
Metastasis to the adrenal gland

The correct answer is choice C.

Myelolipomas of the adrenal gland and up to 60% of adrenal adenomas (choice C) will generally have lipid content within that can place Hounsfield
density unit measurements in the 8-12 range, as in this case. In general, although with exceptions, adrenal masses < 4 cm in size with < 10
Hounsfield units in density tend to be benign lesions. Adrenal adenomas are the most common benign lesions of the adrenal. Most adenomas are
small (< 5 cm) and nonsecretory, and of those that are secretory, the majority are cortisol secreting. Myelolipomas are also small benign tumors of
the adrenal that contain lipid and hematopoetic tissue. They are functionally inactive and of little clinical significance unless they become large and

exert mass effect, which is quite rare.

Myelolipoma alone (choice A) would demonstrate Hounsfield density unit measurements in the 8-12 range, but so could up to 60% of adrenal
adenomas, so choice C is the correct answer.

In contrast, pheochromocytomas (choices B & D), adrenal carcinoma, and metastasis to the adrenal gland (choice E) are all malignant processes
that characteristically do not demonstrate low Hounsfield unit density measurements on noncontrast enhanced CT scans as the other conditions
listed do.

A 68 year old male presented to the emergency room with mid-abdominal and back pain, a sensation of fullness in his abdomen, and fatigue. On
physical examination his blood pressure was 98 over 60 and he was found to have a pulsatile mass in his mid abdomen. His hematocrit came back
with a value of 26. The emergency room physician asked the patient if he had noticed any blood in his stool and the patient replied that he had
seen some dark, reddish black blood in his stool intermittently over the last 3 days. The patient is then sent to the radiology department for a CT
scan of his abdomen, which is performed with intravenous contrast, but without oral contrast. A mid-abdominal image from this study is shown.
Based on this CT image, which of the following is the correct diagnosis?

1)
2)
3)
4)
5)

Abdominal aortic aneurysm (AAA)


Leaking abdominal aortic aneurysm
Aortico-enteric fistula (AEF)
Hemoperitioneum
All of the above

The correct answer is choice E.

All of the above diagnoses (choice E) are applicable to this patient.

The CT image through the mid abdomen demonstrates an abdominal aortic aneurysm (choice A) with mural thromubus. In addition, there is illdefined soft tissue streaking of peri-aortic and intra-abdominal fat, consistent with leaking of the aneurysm (choice B) and hemorrhage into the
retroperitoneum and peritoneal cavity. Free fluid, presumed to be blood (choice D), is present, seen best in the left paracolic gutter. Of particular
note, bearing in mind that this patient did not receive oral contrast, there is a significant amount of contrast material seen within the lumen small
bowel loops on the left side of the abdomen, as well as in the 4th portion of the duodenum. These findings are consistent with and aorticoduodenal

fistula (choice C).

Abdominal aortic aneurysms (AAA) are not uncommon sequelae of atheroscleriotic disease. AAA is defined as a greater than 3 cm focal widening
of the abdominal aorta. There is a male to female predominance of 5:1. Many patients with small AAAs are asymptomatic. The larger the AAA
becomes, the more likely that the patient will become symptomatic with awareness of a pulsatile abdominal mass and possible abdominal pain. The
larger an AAA becomes, the more likely that it will rupture. An AAA with a diameter of 8 cm has an extremely high risk of rupture.

Aortico-enteric fistulas (AEF) are fairly rare and may be classified as primary or secondary. Primary AEF, as in the case presented, is most
frequently due to atherosclerosis of the aorta. The most common location (88%) for the AEF is the 3rd and 4th portions of the duodenum.
Gastrointestinal hemorrhage hematmesis or melena, may be the first symptom of an AEF. Many AEFs are associated with back pain as well.
Secondary AEF may result from trauma or iatrogenic causes such as erosion of a suture line. AEF carries high morbidity and mortality rates so
early diagnosis is essential for improved outcome.

You arrive early on the first day of your CT scan rotation as a second year resident. There are a number of CT scans that were done on outpatients
the night before which now need to be read. You are surprised when the first one you pull up shows the attached image. You dig into the patients
medical record and find that he is a 52 year old male with back pain, hematuria, hypertension, and a history of frequent urinary tract infections. You
make the diagnosis of adult polycystic kidney disease.
This disease is associated with all of the following conditions except for:

1)
2)
3)
4)
5)

Autosomal recessive inheritance


Hepatic cysts
Pancreatic cysts
Urolithiasis
Berry aneurysms

The correct answer is choice A.

Adult polycystic kidney disease has an autosomal dominant mode of inheritance. Infantile polycystic kidney disease has an autosomal recessive
mode of inheritance (choice A).

Adult polycystic kidney disease affects 1:6000 people and accounts for up to 15% of patients on dialysis in the U.S. Symptoms such as hematuria
and flank pain usually begin between 30 and 50 years of age. 80% of such patients have associated hypertension. Urolithiasis (choice D) occurs in
up to 30% of patients afflicted. Hepatic cysts (choice B) occur in approximately 60% of patients and are seen on the CT scan here. Pancreatic cysts
(choice C) are present in 10% of patients. Finally, berry aneurysms (choice E) are present in up to 40% of these patients, and their rupture (9%)
may cause of death.

During a routine visit to his family physician, a 51 year old male states that he has low back pain and stiffness, which seems to have come on and
progressed over the past 6 months or so. Review of systems reveals mild anorexia and a 5 lb weight loss over the same time duration. Physical
examination demonstrates limited mobility of the lower spine, but is otherwise unremarkable. Screening blood tests are only significant for a
moderately elevated ESR. The patient is sent to radiology for lumbar spine X-rays, of which the PA view is shown here.
What is the most likely diagnosis in this patient?

1) Diffuse Idiopathic Skeletal Hyperostosis (DISH)

2)
3)
4)
5)

Rheumatoid arthritis
Psoriatic arthritis
Anklyosing spondylitis
Reiters syndrome

The correct answer is choice D.

Ankylosing spondylitis (AS) is a chronic inflammatory disorder that affects mainly the axial skeleton. It affects men more than women at a ratio of
15:1. 95% of AS patients are HLA-B27 positive. Onset is usually insidious and frequently presents at ages 15 35 years. Complaints are frequently
of lower back pain and stiffness, with sacroiliac pain as well. Peripheral joint manifestations of the disease can occur in up to 20% of patients. Iritis
may also occur in up to 20% of patients. AS can also be associated with pericarditis, aortic insufficiency, pulmonary fibrosis, and inflammatory
bowel disease. AS affects synovial and cartilaginous joints, as well as sites of tendon and ligament attachment to bone. Radiologic findings include
changes involving the sacro-iliac joints (SI joints) and spine most commonly. SI joint changes are generally symmetric, predominate on the ileal
side of the joint at the middle and lower portions of the joint, and involve loss of definition of the joint, focal sclerosis, later widening of the SI joint,
and eventual fusion of the SI joint. Radiologic changes in the spine include straightening and squaring of the anterior vertebral body margins,
sclerosis of the vertebral body corners, ossification of the annulus fibrosus, marginal syndesmophyte formation, disco-vertebral erosion and
destruction, and eventual bamboo spine due to the undulating contour of ligamentous calcifications and syndesmophytosis. Ankylosis of the
vertebral bodies and apophyseal joints may eventually occur. The anklyosed and bambooed spine may be prone to fracture, resulting in a
pseudoarthrosis. Atlantoaxial subluxation may also occur.

The X-ray in this case shows fusion of the SI joints with areas of sclerosis inferiorly and the classic appearance of a bamboo spine due to the
undulating contour of ligamentous calcifications and syndesmophytosis. The dagger sign is also present where ossification of the posterior
spinous ligaments produces a central vertical radiodense stripe, best seen here from L3 L5. On a lateral lumbar spine film of a different patient
with AS, straigtening and squaring of the anterior vertebral body margins is seen here:

DISH (choice A) is characterized by the presence of flowing calcification and ossification along the anterior aspect of vertebral bodies, preservation
of intervertebral disc height, and absence of SI joint changes, making this an incorrect choice.

When rheumatoid arthritis (choice B) affects the SI joints, it is usually asymmetric or unilateral. Rheumatoid arthritis also has a propensity for the
cervical spine and not the lumbar spine. These factors render this choice incorrect.

Psoriatic arthritis (choice C) and Reiters syndrome (choice E), when involving the spine, produce bulky, asymmetric calcified and bony outgrowths
which are separated from the vertebral bodies and intervertebral discs, and therefore these choices are incorrect in this case.

A 28 year old male who was thrown from his motorcycle after having been hit by an automobile is brought into the emergency room via ambulance.
Upon arrival he is complaining of right back pain. Vital signs show a blood pressure of 95/60, and a heart rate of 138. Cursory physical examination
shows abrasions and contusions of the right flank and right side of the mid torso with mild tenderness on palpation of the abdomen. Stat CBC which
comes back while the patient is undergoing an emergency body CT scan reveals a hematocrit of 30. You are interpreting that emergency body CT
scan which is performed during the administration of intravenous contrast, an image of which is shown here.
Based on this CT image through the upper abdomen, which of the following is the most accurate description of this patients condition?

1)
2)
3)
4)
5)

Hepatic hematoma
Subcapsular hepatic hematoma
Hepatic laceration
Hepatic laceration with hemoperitoneum
Hepatic laceration and hematoma with active hemorrhage and hemoperitoneum

The correct answer is choice E.

The CT image of the upper abdomen shown demonstrates a laceration to the right lobe of the liver and a small hematoma within the laceration
posteriorly. There is also evidence of hemopertioneum posterior to the liver, and evidence of active hemorrhage with high density intravenous
contrast material leaking into the laceration anterolaterally, posteriorly within a small round hepatic hematoma, and out into the free
hemoperitoneum. Therefore choice E is the most accurate description of this patients condition.

Abdominal organs may frequently be injured by blunt or penetrating trauma blunt trauma in the form of motor vehicle accidents or falls, and
penetrating trauma in the form of stab wounds or gunshot wounds. CT scanning with intravenous and oral contrast is the ideal imaging modality for
evaluation of traumatic injury to the abdomen. In the evaluation of liver trauma, CT is effective for the detection of hepatic lacerations, hepatic
hematomas, active hepatic hemorrhage, and the presence of hemoperitoneum. The presence of active hepatic hemorrhage and or
hemopertoneum are important distinctions to be made as they require interventional management. Hepatic lacerations and hepatic hematomas that
remain within the liver capsule may be managed conservatively, and followed with serial CT scans.

Hepatic lacerations are seen as linear interruptions in normal liver parenchyma that have irregular margins and contain blood. They may even take
on a branching or stellate pattern. Hepatic hematomas are collections of blood within the liver, perhaps within the laceration, that take on a round or
oval shape. Subcapsular hematomas appear as a crescent or oval-shaped lesion immediately deep to the liver capsule which has mass effect and
indents the normal liver tissue. Extravasation of intravenous contrast material into a liver laceration or into the peritoneal cavity is seen as high
density contrast material with Hounsfield units measurements of 30 or greater in the laceration or extending into the peritoneal cavity, indicating
active hemorrhage.

Hepatic hematoma (choice A) is present in this patient and seen as a small, rounded collection of blood in the inferior aspect of the hepatic
laceration, but does not give a full accurate description of this patients condition and is therefore an incorrect answer.

Subcapsular hepatic hematoma (choice B) is an incorrect choice in this patient as this condition would manifest as a crescent or oval-shaped lesion
immediately deep to the liver capsule, having mass effect and indenting normal liver tissue not seen in this case.

Hepatic laceration (choice C) and hepatic laceration with hemopertioneum (choice D), while both present in this patient, still do not give a complete
accurate description of the findings on the CT scan leaving out the active hemorrhage component, and are therefore incorrect answers.

You are reading a chest X-ray on a 32 year old black female that has come in through the emergency room. The requisition for the CXR has the
typical rule out pneumonia filled in as the reason for the examination. On the CXR you see mid lung zone interstitial lung disease with minimal
bilateral hilar lymphadenopathy. On a comparison CXR from two years ago, you see moderate bilateral hilar lymphadenopathy, but without
associated interstitial lung disease.
The most likely diagnosis in this patient is:

1)
2)
3)
4)
5)

Hodgkins disease
Lymphangitic metastases
Reactivation tuberculosis
Sarcoidosis
Pneumocystis carinii pneumonia (PCP)

The correct answer is choice D.

Sarcoidosis (choice D) is the correct answer as this disease most commonly affects black females between the ages of 20 40 years, and has as
one of its hallmarks the progression of interstitial lung disease with a reduction in hilar lymphadenopathy over time. Classic egg-shell calcifications
may also be seen in approximately 5% of patients with long-standing sarcoidosis.

Other interstitial lung diseases may also be associated with bilateral hilar lymphadenopathy, but in the following cases that were offered as choices,
the interstitial lung disease is generally in a different distribution than that of sarcoidosis. Lymphangitic metastases (choice B) may have associated
interstitial lung disease that would generally be located in the lower lung zones and reactivation tuberculosis (choice C) may have associated
interstitial lung disease that would generally be located in the upper lobes. Both choices B & C, as well as choice A Hodgkins disease, would not
show the hallmark of sarcoidosis described a progression of interstitial lung disease with a reduction of hilar lymphadenopathy over time.

Pneumocystis carinii pneumonia or PCP (choice E) frequently causes a diffuse linear reticular interstitial lung pattern early on in its process, but is
not typically associated with bilateral hilar lymphadenopathy.

A 20 year old male jogger was out for his morning run when at 3 miles he stumbled on the trail and twisted his right ankle. He presents to the
emergency room with complaints of pain and swelling of his right ankle. Physical examination reveals point-tenderness over the distal right fibula
with soft-tissue swelling and bruising in the area. The patient is sent to radiology for X-rays of his right ankle, an image of which is shown here.
With regard to bone fractures, in addition to examining X-rays for breaks in continuity of bone cortex and radiolucent fracture lines, which of the
following should also be sought?

1) Overlap of cortical and spongy bone creating an abnormally white zone

2)
3)
4)
5)

Unexplained fragments of bone even in the absence of a visible fracture


Dense areas of bone seen in 2 views where impaction has occurred
Flocculent calcific density in soft tissue adjacent to bone
All of the above

The correct answer is choice E.

In addition to examining X-rays for breaks in continuity of bone cortex and radiolucent fracture lines, all of the above (choice E) radiographic signs
should be looked for in the evaluation of bone fractures.

Choices A C above are self-explanatory. Flocculent calcific density in soft tissue adjacent to bone (choice D) signifies callus formation in a healing
bone fracture and indicates that a subacute fracture is present.

The X-ray shown demonstrates a spiral fracture of the distal right fibula with minimal lateral displacement of the distal fragment and overlying softtissue swelling.

The descriptions of fractures on X-ray are important so as to convey specific information to the orthopedist. Fractures may be transverse, oblique,
or spiral in orientation. They may be complete having broken through both cortical surfaces of the bone, or incomplete having broken through
only one surface of bone. Incomplete fractures, often called greenstick fractures are common in growing children whose bones contain more
fibrous material than those in adults. Fractures may be simple with no bony fragment(s) involved, or comminuted with bony fragments involved.
Compound fractures describe fractures in which bone has broken through the skin surface. An avulsion fracture refers to a fracture in which a small
fragment of bone is traumatically pulled off a bone by a tendon or ligament at its insertion site. Intra-articular fractures describe fractures that extend
into joint spaces. A stress fracture may result when there is abnormal stress exerted on bone. Fatigue fractures are stress fractures that occur
when abnormal stresses are applied to normal bone, such as in the case of march fractures in the metatarsals of military recruits. Insufficiency
fractures are stress fractures that occur when normal stresses are applied to abnormal bone, such as in the case of a compression fracture of a
vertebral body in an older woman with osteoporosis. A pathologic fracture is a descriptive term reserved for a fracture in an abnormal bone that is
rendered soft, such as in the case of bone cysts or in Pagets disease.

In addition to the descriptors listed above, it is important to also convey the location of the fracture, its alignment, degree of angulation of the distal
fragment, and amount of override if any is present.

If there is a high clinical suspicion of a bone fracture, but there is no evidence to support it on plain X-ray, a CT scan should be performed of the
area in question as this imaging modality is far more sensitive in the detection of fractures than plain radiography.

A 62 year old male presents to the emergency room with complaints of sharp right upper quadrant pain that comes in waves and lasts for hours. He
has been experiencing this pain intermittently over the last 2 days as well as a loss of appetite and general malaise. Physical examination reveals a
temperature of 100.3, RUQ tenderness and guarding, and no evidence of jaundice. The ER physician sends the patient to radiology for a RUQ
ultrasound, which you are interpreting. An image of that ultrasound is shown.
What is your leading diagnosis?

1)
2)
3)
4)
5)

Acute cholecystitis
Chronic cholecystitis
Acalculous cholesystis
Emphysematous cholecystitis
All of the above

The correct answer is choice A.

The ultrasound shows a longitudinal image through the gallbladder which contains multiple nonechoic gallstones mixed with sludge. There is
thickening of the gallbladder wall with a measurement of > 3 mm. There is also a hazy delineation of the gallbladder wall consistent with its
inflammation. These ultrasonographic findings combined with the patients history suggest a diagnosis of acute cholecystits (choice A).

Other important ultrasonographic findings in acute cholecystitis are a positive ultrasound Murphys sign which refers to the elicitation of pain when
the ultrasound probe is pressed against the gallbladder. The presence of pericholecystic fluid is another frequent sign found in acute cholecystitis.
The gallbladder wall may demonstrate a linear lucency due to edema within, producing the halo sign, which is also associated with acute
cholecystitis. Frequently, gallstones present will be calcified and therefore echogenic with limited through transmission.

Cholecystitis occurs more frequently in females than males at a ratio of 3:1, peaking in the 4th 6th decades of life. The classic mnemonic is the 4
Fs Female, Fat, Forties, and Fertile.

90% of acute cholecystits is secondary to cystic duct obstruction by an impacted calculus with secondary chemical irritation to the gall bladder wall
from concentrated bile, possible bacterial infection, and reflux of pancreatic secretions if a calculus becomes lodged distally in the sphincter of
Oddi.

Chronic cholecystitis (choice B) does not fit the clinical history of this patient. The ultrasound appearance of chronic cholecystitis would likely show
a small, contracted gallbladder with a thickened wall, containing calculi. Calcification of the gallbladder wall can occur in this condition forming a
porcelain gallbladder.

Acalculous cholecystitis (choice C) is not common and probably occurs secondary to decreased blood flow through the cystic artery for a variety of
reasons. The patient presented here demonstrated gallstones on the ultrasound so this choice does not apply.

Emphysematous cholecystitis (choice D) refers to a severe form of cholecystitis that most frequently occurs in diabetic males. Ultrasound would
demonstrate high levels of linear echogenicity outlining the gallbladder wall due to the gas contained within the wall. This condition carries a risk of
gangrene and perforation of the gallbladder.

A 27 year old female who was walking in a somewhat dangerous part of town late at night is brought into the emergency room via ambulance after
having been stabbed during a mugging in which the thief made off with her purse. The young woman arrives in severe respiratory distress with a
blood pressure of 90/60, heart rate of 120, and respiratory rate of 40. The clothing on the left side of her body is bloody. Cursory physical
examination reveals decreased breath sounds on the left. A portable chest x-ray is immediately obtained and that image is shown here.
What is your primary diagnosis in this patient?

1)
2)
3)
4)
5)

Pneumothorax
Tension pneumothorax
Hemothorax
Lung contusion/injury
Right lung volume loss with ipsylateral shift of mediastinum

The correct answer is choice B.

This patients diagnosis is tension pneumothorax (choice B) based on the chest x-ray findings of a large left-sided pneumothorax with mediastinal
shift to the right, or contralateral side.

Tension pneumothorax is a serious, life-threatening condition. It is associated with a ball-valve type mechanism in which increased volume of air
and pressure accumulates in the pleural space so as to cause shift of the mediastinum to the contralateral side. This mediastinal shift results in

compression of the right side of the heart, superior and inferior vena cava, thereby limiting blood return to the heart, resulting in cardiovascular
collapse. In addition, loss of volume of the compressed lung on the side of the pneumothorax combined with loss of volume of the lung on the
contralateral side due to compression from mediastinal shift leads to compromise of respiratory function. The most common causes of tension
pneumothorax are iatrogenic causes such as central venous catheter placement and barotrauma due to mechanical ventilation, and traumatic
causes such as penetrating (as in the case presented here) or blunt injuries to the chest, including chest compressions performed during CPR. In
addition to the chest x-ray findings described above, another potential and frequent radiographic sign of tension pneumothorax is depression of the
hemidiaphragm on the ipsylateral side. Treatment of tension pneumothorax involves decompression of the pleural space on the side of the
pneumothorax which may be immediately achieved by needle decompression, and maintained further by placement of a chest tube on the affected
side.

Pneumothorax (choice A) is an incorrect answer in this instance. Although the patient technically does have a pneumothorax on the left, it is of
extreme importance that the distinction between a pneumothorax and tension pneumothorax be readily made as the latter is a serious, lifethreatening situation that requires immediate intervention.

Hemothorax (choice C) is also an incorrect answer in this instance. While there may be a small amount of fluid in the left pleural space in this
patient, most likely to be blood due to the patients history, this is not the primary and emergent diagnosis in this patient for reasons stated above.

Lung contusion/injury (choice D), again while possible in this patient, it is not the primary and most emergent diagnosis, and is therefore an
incorrect answer.

Right lung volume loss with ipsylateral shift of mediastinum (choice E) is also an incorrect answer. While there is shift of the mediastinum to the
right with secondary volume loss on the right, the mediastinal shift is due to the presence of the tension pneumothorax on the left, the far more
emergent diagnosis.

A 28 week premature male neonate who is now 2 weeks old has been in the neonatal intensive care unit since birth and was started on tube
feedings of his mothers breast milk 3 days ago. The neonate has begun spitting up the breast milk, has a distended abdomen, and traces of blood
in his stool. The pediatrics resident is concerned that the patient may have necrotizing enterocolitis (NEC) and orders a portable abdominal X-ray,
which you are called up to the nursery and asked to review with the resident.

Which of the following radiographic findings on the abdominal X-ray should you be looking for when reviewing the abdominal film?
1)
2)
3)
4)
5)

Dilatation of bowel loops


Pneumatosis intestinalis
Portal venous gas
Pneumoperitoneum
All of the above

The correct answer is choice E.

All of the above (choice E) - dilatation of bowel loops (A), pneumatosis intestinalis (B), portal venous gas (C), and pneumoperitoneum (D), are all
radiographic signs that may be present on abdominal X-ray in cases of NEC.

Necrotizing enterocolitis (NEC) is an acute, life-threatening abdominal condition of the neonate seen more frequently in premature infants of low
and very low birth weight. NEC is thought to be due to intestinal mucosal damage, ischemia and necrosis. The loss of intestinal mucosal integrity
allows bacteria to enter the bloodstream and establish the potential for sepsis.

NEC occurrence is inversely proportional to gestational age and affects 10% of neonates with birth weights under 1500 grams. Symptoms may
include feeding intolerance, vomiting, diarrhea, blood in stool, and signs of abdominal distension and temperature intolerance may be present.

Early diagnosis and treatment with bowel rest, hydration and antibiotics is paramount to decreasing morbidity and mortality. The mainstay of
diagnosis for NEC remains the plain abdominal X-ray with a cross-table lateral film to rule out pneumoperitoneum. Plain film radiographic findings in
NEC include dilatation of bowel loops (choice A) seen in nearly all cases, which is manifestation of ileus and a nonspeicific finding for NEC.
Pneumatosis intestinalis (choice B) is a specific radiographic sign for NEC which may actually occur before clinical signs become present. Note,
however, that the absence of pneumatosis intestinalis does not rule out the possibility of NEC. Portal venous gas (choice C) is a later radiographic
sign in NEC due to an extension of intramural gas that enters the veins of the bowel wall and extends to the portal vein. Pneumoperitoneum (choice
D) results from bowel perforation and is an accepted radiographic indication for surgical intervention.

Other potential radiographic signs are bowel wall thickening and intraabdominal fluid, both of which are better evaluated with abdominal ultrasound.

Another distinct use for abdominal ultrasound in NEC is for the assessment of bowel wall perfusion.

An approximately 34 year old homeless man is brought via ambulance into the emergency room by EMTs who were called to pick him up because
he was found to be unconscious. Nothing more is known about this patient. The ER physician finds the patient comatose with borderline vital signs
and without any obvious signs of trauma. He sends the patient to radiology for a CT scan of his head which you initially perform without intravenous
contrast. On the unenhanced head CT scan you identify a high density mass-like region intracranially in the region of the right sylvian fissure.
All of the following conditions can demonstrate high density intracranial lesions on unenhanced CT scan of the head except for:
1)
2)
3)
4)
5)

Meningioma
Metastasis
Ischemic cerebral infarction
Primary lymphoma
Intracerebral hemorrhage

The correct answer is choice C.

Ischemic cerebral infarction (choice C) due to occlusion of a cerebral artery classically produces a triangular or wedge-shaped area of hypodense
cortex in the vascular distribution of the occluded artery on head CT scans. In the case of ischemic cerebral infarction, there may be peripheral
enhancement of the infarct with intravenous contrast administration.

Meningiomas (choice A) are well demarcated, often rounded lesions located adjacent to dura which are frequently hyperdense on unenhanced
head CT scans due to the presence of psammomatous calcifications. Menigiomas characteristically demonstrate intense homogeneous
enhancement with intravenous contrast administration. Certain mestastases (choice B) such as that of osteosarcoma and some colon cancers, as
well as hemorrhagic metastases, can be very dense on unenhanced head CT scans. Primary lymphoma (choice D) can also be hyperdense on
unenhanced head CT scans and tends to enhance intensely with intravenous contrast administration. Finally, intracerebral hemorrhage (choice E)
is characteristically hyperdense on unenhanced head CT scans and may be the result of trauma, aneurysm rupture, hypertensive vascular disease,
or iatrogenic causes. The following is a CT image showing traumatic intracerebral hemorrhage of the frontal lobes bilaterally:

An 11 year old girl is brought to her pediatricians office by her mother who says that the child is complaining of discomfort when urinating. The girl
also complains of occasional lower abdominal pain. Physical examination is unremarkable except for a temperature of 100.1. The pediatrician has
the patient collect a urine specimen in the office and he can see that the urine is cloudy, but without visible blood. A quick urinalysis shows multiple
WBCs in the urine. For further evaluation, the pediatrician sends the patient to radiology for an intravenous pyelogram (IVP) to be followed by a

renal ultrasound. You are interpreting the IVP, a film from which is shown here.
Which of the following conditions is the primary diagnosis that you will assign to this patient?

1)
2)
3)
4)

Cross-fused ectopia
Horseshoe kidney
Pelvic kidney
Renal agenesis

5) Renal malrotation
The correct answer is choice C.

Ectopic kidney, or pelvic kidney in this case (choice C) is a congenital anomaly of positioning of the kidney due to the metanephros retaining its
pelvic position and failing to ascend normally into the upper retroperitoneum during fetal development. It has an incidence of about 1:900. Pelvic
kidneys frequently do not undergo the normal internal rotation in development and are found positioned with their ureters projecting anteriorly.
Pelvic kidneys retain their fetal blood supply, derived from the iliac arteries. In many cases of pelvic kidney, patients are asymptomatic and the
diagnosis is not made until some imaging study is performed for another reason. Females with asymptomatic pelvic kidney may present in late
pregnancy with complaints of pain and urinary symptoms due to mechanical pressure of the fetus on the pelvic kidney. As in the case presented
here, patients with pelvic kidney can also present with urinary tract infections, which are often recurrent. In other cases still, these patients may
present with urolithiasis, pelvic pain, or detection of a pelvic mass on abdominal examination. When renal ectopia is diagnosed, it is important to
establish the presence of the contralateral kidney. On the film presented here, there is partial visualization of the right ureter just lateral to L5 and
just lateral to the urinary bladder in the pelvis, so it is known that the right kidney is present and confirmed to be in normal position on other films
from the IVP.

Cross fused ectopia (choice A) is another congenital anomaly of renal development in which the developing kidneys fuse and one ascends with the
other, both on one side. Based on findings on the film presented, this choice is incorrect.

Horseshoe kidney (choice B) is yet another congenital anomaly of renal development in which the lower portions of the developing ascending
kidneys make contact and fuse, then ascend together as one horseshoe shaped kidney, halting at the level of the inferior mesenteric artery which
prevents further ascent. Again, based on findings on the film presented, this choice is incorrect.

Renal agenesis (choice D) is not applicable here as there are 2 kidneys present.

Renal malrotation (choice E) does apply to the pelvic kidney that is present, but is not the primary diagnosis.

A 42 year old woman is sent to the radiology department by her general practitioner for her fist screening mammogram. She has a positive family

history of breast cancer her mother and maternal grandmother having been afflicted. The mammogram reveals a 1.2 cm mass in the upper outer
quadrant of the right breast that is somewhat lobulated and has indistinct margins with a small area of spiculation along one margin. The mass
does not contain any calcifications. You are suspicious that the mass may be malignant and recommend that it be biopsied.
With regard to mammographic findings indicative of malignancy, which of the following findings is considered to be the most highly suggestive for
malignancy?
1)
2)
3)
4)
5)

A mass with indistinct margins


Microlobulation of a mass
Architectural distortion
Spiculation of a mass border
Greater than 5 clustered microcalcifications

The correct answer is choice D.

A mass with a spiculated border (choice D) is considered the most highly suggestive mammographic sign for malignancy. Strands of fibromalignant
tissue can radiate out from a malignant mass resulting in a spiculated, stellate appearance of the mass on mammography. The spiculation
represents fibrosis related to a desmoplastic reaction elicited by some cancers. A post-surgical scar may also produce a spiculated appearance in
the breast, but history and stability over time would aid evaluation in this instance. A spiculated mass that is clearly not a post-surgical scar should
be considered malignant.

A mass with indistinct margins (choice A), a mass with microlobulation (choice B), architectural distortion (choice C) and greater than 5 clustered
microcalcifications (choice E) should all raise concern when interpreting mammograms, but none of those mammographic findings are diagnostic of
malignancy and can be seen in benign disease processes of the breast.

A 56 year old man comes into the emergency room complaining of fever and intense mid-abdominal pain that is radiating to his back. On physical
examination his mid abdomen is tender to palpation and there are diminished bowel sounds. Serum amylase and lipase are elevated. The ER
physician concludes that the patient has pancreatitis and sends him to radiology to have a CT scan of his abdomen for evaluation.
Which of the following radiographic findings would you not expect to see on CT scan in a patient with acute pancreatitis?

1)
2)
3)
4)
5)

Normal appearing pancreas


Colon cut-off sign
Enlarged pancreas with convex margins
Pancreatic psuedosyst formation
Pancreatic calcifications

The correct answer is choice E.

The presence of pancreatic calcifications (choice E) suggests chronic pancreatitis and such a finding is not characteristic of acute pancreatits.

There are many potential etiologies for acute pancreatitis including gallstone pancreatitis due to obstruction of the pancreatic duct by a gallstone,
alcoholic pancreatitis, drug induced, iatrogenic following ERCP, infectious, metabolic, and idiopathic.

Early on in the disease process of acute pancreatitis, the pancreas can appear normal (choice A) when imaged in up to 25% of cases. The colon
cut-off sign (choice B) represents functional spasm of the distal transverse colon as a response to the adjacent inflamed pancreas with gas
proximal, but not distal to the spastic segment. The colon cut-off sign is more commonly associated with plain X-rays of the abdomen, but can be
seen on CT scan as well. Enlargement of the pancreas with convex margins (choice C) due to edema is a frequent finding in acute pancreatitis, as
is pseudocyst formation (choice D) depending on the severity of the disease.

A mother brings her 6 year old son to the pediatricians office because the boy has been complaining of pain in the region of his right lower shin for
2 days after falling off his bicycle and scraping the skin in that area. The boys mother also tells the pediatrician that the child has been running a
low-grade fever all day. Physical examination reveals point tenderness in the region of the shin scrape which seems out of proportion to the degree
of the wound. The patient is sent for X-rays of the right tibia/fibula which were interpreted as normal. With the history of fever the pediatrician is
concerned about the possibility of osteomyelitis and orders a bone scan for further evaluation.
Which of the following statements regarding nuclear bone scintigraphy is true:
1)
2)
3)
4)

Bone scintigraphy can detect osteomyelitis up to 2 weeks before plain X-rays show findings
False negative technetium-MDP scans can occur in children early on in the disease process
Technetium-MDP, Gallium, and In-111 labeled WBC's can all be used in bone scintigraphy
A history of trauma decreases the specificity of a bone scan for osteomyelitis

5) All of the above statements are true


The correct answer is choice E.

Bone scintigraphy is extremely sensitive for osteomyelitis and can demonstrate focal areas of isotope uptake up to 2 weeks before plain film X-rays
may manifest findings such as periosteal elevation (choice A).

False negative Technetium-MDP scans can occur in children early on in osteomyelitis (choice B) due to an early ischemic phase of the disease
prior to the hyperemic phase. In such cases, if there is a high clinical suspicion of osteomyelitis, the technetium-MDP scan should be repeated in 23 days, or a Gallium scan performed which can be positive as early as 4 hours after the onset of the infection. A negative Gallium scan virtually
rules out the possibility of osteomyelitis.
Technetium-MDP, Gallium, and In-111 labeled WBC's all can be used in bone scintigraphy (choice C) for varying reasons. As mentioned, Gallium
scanning has a high sensitivity very early on in osteomyelitis. In-111 labeled WBC scanning is felt to be superior to bone/Gallium imaging in the
detection of infection at sites of fracture non-union.

A history of trauma does in fact decrease the specificity of a bone scan for osteomyelitis (choice D) as any insult to bone may cause isotope
accumulation in the area.

An 84 year old male lost consciousness while at home playing cards with his wife and he is brought to the emergency room via ambulance and
arrives intubated. His wife tells the ER physician that just before he lost consciousness, he had a glazed look, had difficulty speaking to her, and
may have had some seizure activity. She also states that the patient is hypertensive and on medication for the hypertension. On physical
examination, the patient has decerebrate posturing and a Glascow Coma Scale rating of 4. He is immediately sent to radiology for a CT scan of his
brain which is performed without intravenous contrast. An image from that brain CT scan is shown.
Which one of the following statements regarding findings on this CT scan is true?

1)
2)
3)
4)
5)

Right-sided anterior cerebral artery and middle cerebral artery infarct with mass effect and shift across the midline
Right-sided middle cerebral artery infarct with mass effect and shift across the midline
Right sided middle and posterior cerebral artery infarct with mass effect and shift across the midline
Massive hemorrhage of the right cerebral hemisphere with mass effect and shift across the midline
Massive hemorrhage of the right cerebral hemisphere with mass effect but no shift across the midline

The correct answer is choice B.

There is an infarct of the right cerebral hemisphere in the distribution of the right middle cerebral artery (MCA) with secondary edema and mass
effect effacing the anterior and posterior horns of the right lateral ventricle with shift of the ventricular structures across the midline (choice B). The
mass effect seen in combination with the patients comatose state suggests compression on the brainstem and probable herniation of brain through
the foramen magnum, which is a life-threatening situation.

The MCA is the largest of the cerebral arteries and the one most commonly affected by stroke. Males are affected more than females with an
incidence ratio of 3:1. Blacks are affected > Hispanics > Caucasians. Risk of MCA stroke increases with increased age, with the highest occurrence
being in the 7th and 8th decades of life. Other risk factors include hypertension and atherosclerosis. Morbidity and mortality is significantly
increased in cases of hemorrhagic stroke. The majority of MCA strokes are due to embolism, with cardoiembolism (associated atrial fibrillation)
accounting for 50% of total MCA strokes. Emboli may also arise from the carotid bifurcation. Occlusion of the entire MCA, as is the case in this
patient, is relatively rare and would require embolic material to be 3-5 mm in size. The MCA supplies the outer convex brain surface, the basal
ganglia, and the posterior and anterior internal capsules on its respective side. Ischemic stroke in the distribution of the MCA can cause a wide
variety of motor, speech, visual and behavioral defects, depending on specific areas affected.

The right sided anterior cerebral artery (choice A) and posterior cerebral artery (choice C) distributions are not affected by infarction in this case,
and therefore choices A & C are incorrect.
Intracerebral hemorrhage appears
as very high density on CT scan and there is no evidence of intracerebral hemorrhage on the CT scan (choices D & E) in this case, therefore
choices D & E are incorrect. Choice E is also incorrect in stating that there is no shift across the midline.

You are moonlighting as a radiology resident interpreting studies for a remote hospital on the night shift. The attached noncontrast enhanced image
is sent to you without anymore history than rule out intracranial hemorrhage.
The most likely diagnosis in this patient is which of the following?

1)
2)
3)
4)
5)

Epidural hematoma
Subdural hematoma
Subarachnoid hemorrhage
Subdural empyema
Intracerebral hemorrhage

The correct answer is choice B.

This image demonstrates classic CT findings in a patient with a subdural hematoma (choice B) there is an extraaxial high density crescentric fluid
collection surrounding the left cerebral hemisphere with extension into the interhemispheric fissure. The collection has a concave inner margin and
convex outer margin. In addition, there is mass effect with effacement of cortical sulci, deviation and compression of the lateral ventricle, and
midline shift.

An epidural hematoma (choice A) is characteristically elliptical in shape with biconcave margins and may also cause mass effect. Distinction
between a subdural and an epidural hematoma is important. A CT scan of an epidural hematoma is shown here:

Subarachnoid hemorrhage (choice C) would show increased fluid density in the cisterns, fissures, sulci and possibly ventricles bilaterally and not
demonstrate mass effect. Subdural empyema (choice D) can be cresentric to lentiform in shape and cause mass effect, but its margins would be

more distinct and would be hypo- or iso-dense with brain tissue on noncontrast CT. Intracerebral hemorrhage (choice E) is an incorrect choice in
this case as the fluid collection shown is extaaxial, not intracerebral.
A 73 year old male presents to the emergency room with complaints of mid abdominal pain which is worse after eating and more severe at night.
The pain has been waxing and waning for 2 days. He states that on the morning of admission, during one of his bouts of pain, his abdomen
became quite bloated and uncomfortable. He denies bright red blood in his stool, but says that his stools have been kind of blackish over the last
couple of days. Past medical history is unremarkable except for a cholecystectomy 16 years previously. On physical examination, his abdomen is
distended and firm. The ER physician sends the patient to radiology for an abdominal X-ray, which is shown here.
What diagnosis do you make on this film?

1)
2)
3)
4)
5)

Small bowel obstruction


Ileus
Small bowel volvulus
Pneumoperitoneum
Pneumoperitoneum and ileus

The correct answer is choice E.

The abdominal film shows definitive evidence of pneumoperitoneum (choice D) with the presence of a positive Riglers sign. Riglers sign occurs
in pneumoperitoneum when gas is present on the inside and outside of bowel wall, allowing visualization of the inner and outer surfaces of the
bowel wall, as in the case shown, particularly well seen at the hepatic flexure of the colon in the right upper quadrant. Another sign of
pneumoperitoneum seen in this patient is the triangle sign where air is situated between 3 loops of adjacent bowel forming a triangle, well seen
here in the right lower quadrant. The "triangle sign" is more commonly seen when the 3 loops of bowel are fluid-filled, and therefore what is seen is
a simply triangular collection of air surrounded by soft tissue density. The 3rd sign of pneumoperitoneum seen on this film is demonstrated in the
right upper quadrant just lateral to T 11 where air is seen on both sides of a portion of the falciform ligament.

The abdominal film also shows multiple moderately dilated loops of both small and large bowel, consistent with ileus (choice B). Small bowel loops
are identified by the visualization of their valvulae conniventes, and large bowel loops are identified by their haustra and positioning.

Since both pneumoperitoneum and ileus are present (choice E), choice E is the correct answer.

It is important to be able to recognize all of the signs of pneumoperitoneum on abdominal films as many patients with this condition are not well
enough to position themselves for upright films the most ideal positioning for identification of pneumoperitoneum.

Signs of pneumoperitoneum are:

1. Free air under hemidiaphragm(s) seen on upright films, particularly well seen on the right side with air between the upper surface of the liver
and under surface of the right hemi-diaphragm.

2. Free air under right hemi-diaphragm over the lateral surface of the liver on a left lateral decubitus film.

3. Riglers sign as discussed above.

4. Triangle sign as discussed above.

5. Falciform ligament sign as discussed above.

6. Football sign where a large amount of free air is positioned anteriorly on a supine film assuming the shape of a football. This sign is
frequently seen in cases of pneumoperitoneum in neonates.

7. Inverted-V sign where air outlines both lateral umbilical ligaments.

8. Urachal sign where air outlines the middle umbilical ligament.

9. There are a multitude of causes of pneumoperitoneum such as any perforated viscus stomach, small bowel, colon, etc.; iatrogenic causes
such as abdominal surgery or endoscopy; gynecological causes such as Rubin test for tubal patency and vaginal douching; intrathoracic
causes such as tracking from pneumomediastinum; and post-traumatic causes.

Small bowel obstruction (choice A) is an incorrect answer as moderately dilated loops of colon are identified on the film ruling out a small bowel
obstruction.

Small bowel volvulus (choice C) is also incorrect as it is difficult to specifically diagnose on plain film, but would present with signs of small bowel

obstruction.

A 35 year old woman with a strong positive family history for breast cancer comes in for her first mammogram, sent by her gynecologist. She states
that she can feel a small lump in the upper outer quadrant of her left breast, and this is confirmed by the gynecologist. You review her mammogram
and there are no suspicious abnormalities detected. You then refer her to ultrasound for further evaluation of the breasts. An image from that
examination demonstrates a lesion in the left upper outer quadrant as shown.
With regard to ultrasound findings in breast cancer, which of the following is not characteristic of malignancy?

1)
2)
3)
4)
5)

Spiculation
Marked hyperechogenicity
Shadowing
Irregular margins
Heterogeneous internal echogenicity

The correct answer is choice B.

The primary use for ultrasound in breast imaging is to differentiate between cystic and solid lesions. If in the case of the patient presented, the
ultrasound had demonstrated a simple cystic lesion, her breast imaging evaluation would have been a BI-RAD 2 benign finding. However, the
ultrasound in this case shows an irregularly shaped, somewhat lobulated, ill-defined hypoechoic mass with mild shadowing. The lesion is clearly not
a simple cyst, and therefore suspicious for malignancy in this patient, and should be biopsied which could be done with palpation.

Although breast ultrasound cannot be used to make definitive suggestions of malignancy, there have been characteristics of malignancy and
benignity observed on ultrasound. Marked hyperechogenicity (choice B) is an ultrasound characteristic nearly always associated with benign
lesions and therefore the correct answer to the question presented here. Other characteristics of benignity are oval shape, sharply marginated, well
circumscribed, homogeneous echotexture, and increased trough transmission.

Breast malignancy, when present on ultrasound, is virtually always hypoechoic. Other findings that may suggest malignancy, but not be diagnostic
of, are irregular margins (choice D), heterogeneous internal echogenicity (choice E), and posterior acoustic shadowing (choice C). Spiculation
(choice A) is also suggestive of malignancy, as it is in mammography.

A 43 year old Jewish male presents to his physician with complaints generalized abdominal pain and cramping, intermittent diarrhea, weight loss,
and anorexia, all going on over the last 2-3 months. Physical examination demonstrates mild diffuse abdominal tenderness, but is otherwise
unremarkable. Hemoccult test is positive for blood in the stool. Screening blood tests show moderate leukocytosis with a left shift and a significantly
elevated ESR. The physician suspects that the patient likely has inflammatory bowel disease and sends the patient to radiology for an abdominal
CT scan, which is performed with intravenous contrast. You are interpreting the study, an image of which is shown here.
Regarding the radiologic manifestations of Crohns disease vs. ulcerative colitis, which of the following statements if false?

1)
2)
3)
4)
5)

Crohns disease can affect any portion of the GI tract whereas ulcerative colitis is limited to the colon
Ulcerative colitis has a continuous distribution whereas Crohns disease can show skip lesions
Crohns disease may demonstrate creeping fat on CT scan whereas ulcerative colitis does not
Ulcerative colitis tends to give rise to abscess formation whereas Crohns disease does not
Crohns disease demonstrates a thicker colonic wall than does ulcerative colitis

The correct answer is choice D.

Choice D is a false statement. In actuality, it is Crohns disease that tends to give rise to abscess formation whereas ulcerative colitis (UC) does
not.

The CT scan shows creeping fat in the right lower quadrant, thickening of the terminal ileum, enhancement of small and right colonic bowel wall,
and abscess formation anteriorly in the right lower quadrant. This patient has Crohns disease.

In addition to the true statements A, B, C & E above, there are many factors, radiologic and otherwise that help to distinguish Crohns disease from
UC. Both diseases cause ulceration of mucosa in their early stages. Crohns disease is a transmural process with granulomatous infiltration

involving the mucosa, muscularis, and serosa, whereas UC is primarily a mucosal process (choice E). Crohns disease can affect any part of the GI
tract from the esophagus to anus, nearly always involves the terminal ileum (with the classic string sign on barium studies including the terminal
ileum), and shows skip lesions with normal GI tract intervening, whereas UC is a continuous process that involves only the colon and extends
proximaly from the rectum toward the cecum (choices A & B). Ulcerative colitis can cause a secondary backwash ileitis when there is extensive
right colonic involvement. Later stages of Crohns disease may show stricture formation in the small or large bowel, whereas UC may show a stiff,
pipe-like colon due to fibrosis. Crohns disease is less likely to show a halo sign of the colon on CT scan than is UC. Crohns disease
demonstrates an irregular outer surface to bowel wall whereas UC tends toward a smooth surface. In the perirectal region, Crohns disease
commonly causes fistula and abscess formation whereas UC demonstrates an increase in size of the presacral space with stranding of fat within.

The following are characteristics of Crohns disease and not of UC: fissure, fistula and abscess formation; fibrofatty proliferation of the mesentery
termed creeping fat, and associated lymphadenopathy.

UC has a high association with carcinoma of the colon and the potential to develop toxic megacolon, both of these conditions can occur with
Crohns disease, but do so with a much lesser frequency.

A 24 year old woman who is on her company softball team was playing out in center field when a high fly ball came her way. She went to catch it,
but the sun was blocking her vision somewhat causing her to fail to make the catch and get hit in the right eye with the softball. She is brought to
the emergency room by a teammate and is complaining of pain on the right side of her face and some double vision. Physical examination reveals
swelling and ecchymosis of the soft tissues below the right eye, and limited orbital mobility on the right with upward gaze. She is sent to radiology
for a CT scan of her face, a coronal image of which is shown here.
What is your diagnosis in this patient?

1)
2)
3)
4)
5)

Tripod fracture
Blowout fracture
LeFort I fracture
LeFort II fracture
LeFort III fracture

The correct answer is choice B.

The CT scan shows a fracture of the floor of the right orbit with mild inferior displacement of the medial aspect of the orbital floor toward the right
maxillary sinuses. There is abundant soft-tissue density (presumably hemorrhage) within the maxillary and ethmoid sinuses bilaterally, right greater

than left.

A blowout fracture (choice B) is a fracture of the floor of the orbit that occurs in cases of blunt trauma to the eye anteriorly (such as in the case of a
flying softball or fist) causing posterior displacement of the orbit. This posterior displacement of the orbit increases pressure in the orbital socket,
which is released at the weakest bony point the orbital floor, which also serves as the roof of the maxillary sinus. The anterior rim of the orbit
remains intact. A depressed orbital floor fracture may entrap the inferior rectus muscle and cause difficulty with upward gaze. Hemorrhage into the
adjacent maxillary sinus is a common associated finding. A blowout fracture by similar mechanism can occur medially involving the lamina
papyracea, but this is far less common.

A tripod fracture (choice A) results from an oblique injury to the face in the region of the malar eminence with resultant fractures at 3 sites: the
frontozygomatic suture line, the zygomatic arch, and the maxilla including the anterior rim of the orbit and lateral wall of the maxillary sinus. Hence
this choice is an incorrect answer.

Lefort fractures result from direct trauma to the anterior face and refer to symmetrical fractures of the facial bones that extend posteriorly to involve
the pterygoid plates.

LeFort I fracture (choice C) is a horizontal fracture of the transverse maxilla across the alveolar ridge and inferior wall of the maxillary sinuses back
to the pterygoid plates which results in a floating palate. Hence this choice is an incorrect answer. A LeFort I fracture diagram is shown here:

LeFort II fracture (choice D), also called the pyramidal fracture, is a vertical fracture through the maxilla, across the nasal bone, and back to the
pterygoid plates resulting in a floating maxilla. Hence this choice is an incorrect answer. A LeFort II fracture diagram is shown here:

LeFort III fracture (choice E) is the most severe of the group with a fracture line that passes through the frontozygomatic sutures, across the nasal
bone, and back to the pterygoid plates resulting in complete craniofacial separation or a floating face. Hence this choice is an incorrect answer. A
LeFort III fracture diagram is shown here:

A 92 year old female nursing home patient is brought into the emergency room via ambulance with symptoms of severe respiratory distress after
having aspirated vomit about an hour after having been fed soup. Upon arrival in the ER, she was intubated and an arterial blood gas was
obtained, which showed a PaO2 of 75%. A chest X-ray was obtained which showed mild bilateral patchy perihilar and basilar infiltrates. The patient
was admitted to the ICU. Over the course of the next 24 hours, the patient becomes increasingly unstable with episodes of hypotension,
temperature instability, and prolonged hypoxemia. The intensivist suspects that the patient may be developing adult respiratory distress syndrome
(ARDS). A portable chest X-ray at 26 hours is shown here.
Regarding the radiographic appearance of ARDS on chest X-ray, which of the following findings would not be expected?

1)
2)
3)
4)
5)

Unremarkable chest X-ray findings within the first 12 hours


Bilateral findings
Kerley B lines
Pleural effusions
Air-bronchograms

The correct answer is choice C.

Kerley B lines (choice C) are not an expected radiographic finding on chest X-ray in ARDS.

ARDS has a multitude of causes including pneumonia, aspiration of gastric contents, pulmonary contusion/trauma, inhalation injury, near drowning,
sepsis, shock, pancreatitis, and many others. The pathopysiology of ARDS involves increased capillary endothelial permeability and alveolar
epithelium damage with resultant high protein fluid in the alveolar space. Patients are hypoxemic and have pulmonary wedge pressures < 18 mm
Hg. Radiographic findings on chest X-ray include possible unremarkable findings within the first 12 hours (choice A), bilateral findings (choice B),
alveolar infiltrates that progress quickly, air-bronchograms (choice E), pleural effusions (choice D), and persistent findings. Kerley B lines (choice C)
and cardiomegaly are not characteristic chest X-ray findings of ARDS. On CT scan, a patchy ground-glass appearance may be present. Therapy
involves treatment of the precipitating cause if identified, and is largely supportive involving mechanical ventilation. Potential complications of ARDS
include pulmonary, mediastinal, and subcutaneous emphysema due to prolonged ventilation; pneumothorax; secondary pneumonia; and possible
resultant pulmonary fibrosis. The mortality rate for ARDS is approximately 35%.

A 42 year old male presented to a walk-in clinic with complaints of dry cough that had been going on for about a month. He has no other specific
complaints, but review of systems revealed that his eyelids tend to feel droopy lately. Physical examination reveals normal breath sounds in the
chest and mild ptosis bilaterally. The patient is sent to radiology for a chest X-ray which shows widening of the superior mediastinum on the right,
and a suggestion of right hilar adenopathy. A chest CT scan is ordered on the patient which is performed with intravenous contrast. An image from
that study is shown.
Of the following mediastinal tumors, which one would you not include in your differential diagnosis for this case?

1)
2)
3)
4)
5)

Malignant thymoma
Lymphoma
Metastatic disease
Thyroid carcinoma
Malignant schwannoma

The correct answer is choice E.

The CT scan image shows a large, bulky, somewhat lobulated, heterogeneously enhancing mass of the left anterior mediastinum. There is also
evidence of bilateral hilar and subcarinal lymphadenopathy.

Malignant schwannoma (choice E) is the only mediastinal tumor listed above that characteristically occurs in the posterior mediastinum, and not the
anterior mediastinum.

The remainder of the choices, malignant thymoma (choice A), lymphoma (choice B), metastatic disease (choice C), and thyroid carcinoma (choice
D) can all occur in the anterior mediastinum and should be listed in the differential diagnosis of this compartment, particularly in the presence of
mediastinal lymphadenopathy.

Differential diagnoses for anterior, middle, and posterior mediastinal lesions are well established. Some of the conditions overlap and may be found
in more than one of the mediastinal compartments. A summary of those differential diagnoses is as follows

Anterior mediastinal lesions the 4 Ts Thymoma, Teratoma, Thyroid tumor/goiter, Terrible lymphoma. Thymoma is the most common tumor of
the anterior mediastinum and anywhere from 30-50% of patients with thymoma have associated myasesthenia gravis. Parathyroid tumors may also
be present in the anterior mediastinum. Lymphadenopathy of the anterior mediastinum may also be present due to metastatic disease as well as
lymphoma. Morgagni hernia is also included in this category as well as aneurysm of the ascending aorta.

Middle mediastinal lesions include causes of lymphadenopathy: lymphoma, metastatic disease, tuberculosis, coccidioidomycosis, sarcoidosis,
and silicosis. Bronchogenic carcinoma and primary tracheal neoplasms may cause middle mediatstinal mass lesions. Other potential middle
mediastinal lesions include foregut duplication abnormalities such as brochogenic cyst and extralobar sequestration. Aneurysms of the aortic arch
or its major branches are also included in this category.

Posterior mediastinal lesions Neruogenic tumors make up the largest group of posterior mediastinal tumors (neuroblastoma, ganglioneuromas,
schwannoma, neurofibroma). Tumors of the spine may present as a posterior mediastinal mass aneurysmal bone cyst, chondrosarcoma,
osteosarcoma, Ewings sarcoma, myeloma and metastasis. Other spine-related conditions to consider are neurenteric cysts, lateral meningoceles,
and estramedullary hematopoiesis. A paraspinous abscess may also present in the posterior mediastinum. Lympadenopathy due to lymphoma or
Castlemans disease may also present here. Aneurysms of the descending aorta and GI abnormalities such as hiatal hernia and esophageal
tumors (cancer and leiomyoma) are included here as well. Foregut duplication abnormalities such as brochogenic cyst and extralobar sequestration
can occur in the posterior as well as the middle mediastinum compartments.

An 82 year old man is sent to the emergency room from his nursing home for intermittent severe lower abdominal pain that is associated with
bloating. Physical examination reveals that the patient is somnolent, slender, afebrile, and has a distended lower abdomen that causes him distress
with palpation. The ER physician sends the patient to radiology for a KUB (plain film of kidneys, ureters and bladder) for further evaluation. You are

interpreting the KUB which is shown.


Of the following choices, what is your working diagnosis?

1)
2)
3)
4)
5)

Pneumopertioneum
Ileus
Diverticulitis
Giant sigmoid diverticulum
Sigmoid volvulus

The correct answer is choice E.

Sigmoid volvulus (choice E) is a condition that afflicts the elderly, in which a redundant loop of sigmoid colon twists on itself at the root of its
mesentery, forming a closed loop obstruction. It tends to occur intermittently and have a high propensity for debilitated and elderly patients. It is the
most common form of volvulus in the GI tract. Predisposing factors include chronic constipation, mobile colon, and megacolon. X-rays obtained

while the volvulus is occurring typically show a distended loop of sigmoid bowel in the lower abdomen with the base of the loop pointing toward the
left lower abdomen. The dilated loop frequently appears as the coffee bean sign with the center groove of the bean being represented by the 2
walls of the sigmoid loops and their intervening mesentery. If the diagnosis is question, a single contrast barium enema may be performed for
further evaluation, which would typically show the bird-beak sign of the barium column as it approaches the twisted pedicle of the distal sigmoid
loop. Failure to make the diagnosis in prolonged cases of sigmoid volvulus may result in ischemia, perforation and peritonitis. Intermittent sigmoid
volvulus is treated with surgical resection of the redundant loop of sigmoid bowel and reanastamosis.

Pneumoperitoneum (choice A) is incorrect as there is no evidence of free intra-abdominal air on the KUB.

Ileus (choice B) is an incorrect choice as in this condition a KUB would show diffusely and evenly distended loops of small and large bowel, as is
not the case on this patients KUB.

Diverticulitis (choice C) frequently demonstrates no findings on KUB, but if present, might show localized ileus, gas in a diverticular abscess,
eccentric mass effect on a loop of colon, or pneumointestinalis. None of these findings are present on this film and therefore this choice is incorrect.

Giant sigmoid diverticulum (choice D) is a rare condition in which a sigmoid diverticulum grows to an extremely large size due to a ball-valve type
mechanism. KUB in this case would show a large, round, gas filled collection generally positioned anterior and in the center of the lower abdomen.
This gas collection would not have the coffee bean appearance with the center ridge as in the case presented, but rather would appear as a
simple gas-filled cyst-like structure in the abdomen.

A 62 year old man presents to his internist with complaints of a dry cough, intermittent fevers and night sweats, a 10 lb weight loss, fatigue and
general malaise, all occurring over the last 2 months. Past medical history reveals that he had stage II colon cancer that was surgically removed 5
years previously. Family history is positive for Osler-Weber-Rendu disease in his father. Review of systems reveals that the patient has never
smoked cigarettes. Physical examination reveals scattered decreased breath sounds of both lungs, but is otherwise unremarkable. The internist
sends the patient to radiology for a chest X-ray, which you are asked to interpret and offer a differential diagnosis of the findings. The PA chest film
is shown.
Which of the following conditions would you include in your differential diagnosis for this patient?

1)
2)
3)
4)
5)

Metastatic disease
Bronchioloalveolar carcinoma (alveolar cell carcinoma)
Lymphoma
Multiple pulmonary ateriovenous malformations (AVMs)
All of the above

The correct answer is choice E.

The chest X-ray shows multiple varying-sized pulmonary nodules bilaterally, with a predominance for the lower lung zones. Some of the nodules
are ill-defined. There are air-bronchograms seen in a large nodule in the left mid lung. There is also a suggestion of a serpiginous, tubular structure
in the left lower lung, and perhaps along the right heart border where there is silhouetting. There is also fullness of the aortico-pulmonary window
suggesting possible mediastinal lymphadenopathy. All these findings present a confusing picture that would include all of the diagnostic possibilities

listed above (choice E) for the following reasons.

Metastatic disease (choice A) should definitely be included in the differential diagnosis for this patient since he has a past history of colon cancer
which can frequently metastasize to the lungs. Other primary cancers that metastasize to lungs are breast, renal, testicular, and melanoma.
Metastasis to the lungs can present as a single pulmonary nodule, multiple pulmonary nodules as in this case, or as lymphangitic spread of
metastasis. Metastasis to the chest can also be in the form of mediastinal and hilar lymphadenopathy.

Bronchioloalveolar carcinoma (choice B) should also be included in the differential diagnosis based on the appearance of the patients chest X-ray.
This tumor is not associated with a history of tobacco use, is slow growing, and may present as a single pulmonary nodule early on with airbronchograms present. It can spread via the airways and manifest as multiple ill-defined nodules in both lungs, as seen in this case. Another
possible presentation is one of local air-space disease. Patients are often asymptomatic, but may present with a dry cough, or in some cases the
cough can produce significant mucoid expectoration. More advanced cases may be associated with weight loss, hemoptysis, chest pain, and fever.

Lymphoma (choice C) is included in the differential diagnosis as it can present in the chest in the form of multiple ill-defined nodules with a
predilection for the lower lobes. A suggestion of AP window lymphadenopathy in this patient would also support this diagnosis. The more frequent
presentation of lymphoma in the chest is one of mediastinal and hilar lymphadenopathy. Parenchymal disease is less common and nearly always
associated with mediastinal and hilar lymphadenopathy.

Multiple pulmonary ateriovenous malformations (choice D) is included in the differential diagnosis of this patient due to the positive family history of
Osler-Weber-Rendu disease in the patients father. This is a hereditary condition in which patients have multiple AVMs that can be located in the
lungs, liver, GI tract, brain and skin. There is also the suggestion on this chest film of the serpiginous, tubular structures as mentioned which may
represent feeding afferent and efferent vessels to and from the AVMs.

A 3 month old male infant is brought in to the pediatricians office by his mother who states that the child seems to tire excessively and have rapid
breathing while feeding. On physical examination, the pediatrician notes that the infant is somewhat small for his age and has a systolic heart
murmur. There is no evidence of cyanosis. The pediatrician sends the child to the radiology department for a single AP chest film and then on to
cardiology for an echocardiogram. You are reading the infants chest film and find mild to moderate cardiomegaly with increased pulmonary
vasculature and an enlarged left atrium.

Which of the following conditions would you include in your differential diagnosis?
1)
2)
3)
4)
5)

Ventricular septal defect (VSD) and atrial septal defect (ASD)


Ventricular septal defect (VSD) and patent ductus arteriosis (PDA)
Endocardial cushion defect (ECD) and atrial septal defect (ASD)
Endocardial cushion defect (ECD) and patent ductus arteriosis (PDA)
Ventricular septal defect (VSD) and endocardial cushion defect (ECD)

The correct answer is choice B.

This patient has signs and symptoms of a left-to-right cardiac shunt without cyanosis, which narrows the differential diagnosis down to all the
congenital heart conditions listed, as well as aorticopulmonary window and partial anomalous pulmonary venous return. Of the choices listed,
choice B ventricular septal defect (VSD) and patent ductus arteriosis (PDA) would both have left atrial enlargement due to increased volume and
pressure in the left atrium. Endocardial cushion defect (ECD) and atrial septal defect (ASD) included in choices A, C, D & E, would not have left
atrial enlargement as the location of the defects tends to relieve pressure within the left atrium, hence no enlargement.

ASD is the most common congenital cardiac lesion, followed by VSD. Note that ECD most often occurs in children with Downs syndrome.

A 58 year old male is sent to the radiology department to have an abdominal CT scan by his urologist due to a finding of occult hematuria. You
perform the CT scan with intravenous contrast and identify a complex 4 cm mass in the left kidney, para-aortic lymphadenopathy, and a filling
defect in the inferior vena cava (IVC).
Which of the following renal tumors is your most likely diagnosis?
1)
2)
3)
4)
5)

Transitional cell carcinoma


Lymphoma
Metastasis to the kidney from a primary lung tumor
Renal adenocarcinoma
Renal oncocytoma

The correct answer is choice D.

A complex mass in the kidney with adjacent lymphadenopathy suggests a malignant process of the kidney with regional metastases. Further, the
finding of a filling defect in the inferior vena cava suggests a diagnosis of renal adenocarcinoma (choice D) as this particular renal tumor has a 30%
incidence of extension into the renal vein and an up to 10% incidence of extension into the IVC. Extension of the tumor may in some instances
reach as far as the right atrium. Patients with renal vein and IVC extension of tumor also frequently have metastases to the lungs as cells break off
from the extended tumor and lodge in the lungs.

The peak age for renal adenocarcinoma is 55 years old, it affects males > females, 2:1, and occult hematuria is a frequent initial presentation.
Renal metastases can also spread to the lungs, liver, bone, and adrenals.

Transitional cell carcinoma (choice A) can occur in the renal pelvis and have regional metastatic lymphadenopathy, but would not characteristically
have renal vein and IVC extension. Likewise, lymphoma (choice B) and metastasis to the kidney (choice C) might also show adjacent
lymphadenopathy, but again, they would not characteristically have renal vein and IVC extension. Renal oncocytoma (choice E) is a benign tumor
of the kidney that would not be associated with regional lymphadenopathy or extension of tumor into the renal vein or IVC.

The most likely cause of the combination of a complex renal mass associated with a filling defect in the renal vein and IVC is renal
adenocarcinoma. However, other rare conditions have been reported to cause similar findings as in a case report describing metastatic melanoma
from the skin to the kidney with extension of tumor into the renal vein and IVC (see suggested reference).

A 61 year old man with complaints of headache and visual changes is sent to radiology for a head CT scan by his neurologist. The CT scan is
performed with the administration of intravenous contrast and you are assigned to interpret the scan. An image from the CT scan is seen attached.
The neurologist calls you to inquire about the findings on the CT scan and asks you for a differential diagnosis.
Your differential diagnosis should include all of the following except for:

1)
2)
3)
4)
5)

Glioblastoma multiforme
Metastatic lesion
Brain abscess
Hydatid disease of the brain
Primary CNS lymphoma

The correct answer is choice D.

The appearance on the contrast enhanced CT scan of the brain is that of a classic ring-enhancing lesion with adjacent intracerebral edema.

Hydatid disease of the brain (choice D) does manifest as a low density round, well demarcated intracerebral lesion, but as a distinction, it is the only
choice listed that characteristically does not show peripheral enhancement and surrounding edema.

The remaining choices listed in the differential glioblastoma multiforme (choice A), metastatic lesion (choice B), brain abscess (choice C), and
primary CNS lymphoma (choice E) all characteristically do demonstrate ring-enhancement and surrounding intracerebral edema.

A previously healthy 72-year-old woman presents to her physician with fatigue, intermittent poorly localized abdominal pain, and discomfort of 10
weeks duration associated with a 20 lb weight loss. The patient has a past medical history of removal of an ovarian tumor 19 years previously.
Physical examination reveals that she is afebrile with abdominal distention and a palpable mass in the mid lower abdomen. Laboratory tests come
back with a serum CA-125 level of 270 U/ml. A CT scan of the abdomen is obtained and an image from the mid abdomen is shown. You are
interpreting the CT scan and offer the following differential diagnosis for the findings to the patients physician.
Which of the conditions listed is the most likely in this patient?

1)
2)
3)
4)
5)

Peritioneal mesothelioma
Tuberculous peritionitis
Lymphoma
Pseudomyxoma pertitonei
Omental cake

The correct answer is choice E.

Omental cake (choice E) is a term used to describe metastatic disease that occurs along the greater omentum, usually as a result of spread of
tumor to the peritoneum. In this case a large, thick, plaque-like mass representing the omental cake is seen against the anterior abdominal wall
which displaces bowel posteriorly. The common malignancies that spread via peritoneal seeding, and thus cause omental caking include ovarian
carcinoma, which is the most common (as in the case of this patient with a past medical history of ovarian carcinoma), as well as gastric, colonic,
and pancreatic cancers. Ascites is frequently present.

Other potential causes of omental caking include peritoneal mesothelioma (choice A) which is associated with a history of asbestos exposure and
has a male to female predominance of 7:1, frequently shows involvement of the liver and colon, the presence of ascites, and would likely have
associated changes of asbestos exposure in the chest as well.

Tuberculous peritonitis (choice B) is another potential cause of omental caking in which involvement of the omentum is frequently more mild, shows
wither a regular or nodular appearance within the peritoneum, a thin omental line covering the omentum, a large amount of ascites, and frequent
calcifications of the liver, spleen and lymph nodes.

Lymphoma involving the peritoneum and omentum (choice C) may also give the appearance of omental caking and would be associated with
ascites and lymphadenopathy in the retroperitoneum and mesentery.

Pseudomyxoma peritonei (choice D) refers to a condition of intraperitoneal spread of a mucin-secreting tumor, usually of the appendix or ovary that
is usually manifested by multiloculated cystic collections throughout the abdomen which displace bowel and produce a scalloping effect on solid
organs within the abdomen.

A 68 year old woman who is one week post-op after a left total hip replacement has the sudden onset of left pleuritic chest pain and intense
shortness of breath. Her arterial blood gas reveals a PaO2 of 65. The orthopedic resident caring for the patient calls you and tells you that he thinks
this patient has had a pulmonary embolism and he asks you for your recommendation on which diagnostic test he should order to make the

definitive diagnosis so he will not risk having to put her on heparin therapy unnecessarily.
You suggest which of the following radiographic studies?
1)
2)
3)
4)
5)

Chest X-ray
Ventilation-perfusion scan (VQ scan)
Transthoracic echocardiography
Spiral CT scan with intravenous contrast
Pulmonary angiogram

The correct answer is choice D.

Pulmonary embolism (PE) is a common and potentially lethal condition. Treated PE has a mortality rate of approximately 5% and untreated PE
mortality rises to 30%. It is therefore quite advantageous to make a rapid diagnosis and begin treatment as soon as possible. Treatment for PE,
particularly in post-operative patients is not without risk, so as in this case, physicians are hesitant to treat patients for PE without a definitive
diagnosis.

Multi-detector spiral CT scanning with intravenous contrast (choice D) is rapidly becoming the test of choice for the diagnosis of PE, has virtually
replaced the VQ scan as the initial investigative study, and nearly eliminated the need for obtaining a pulmonary angiogram, which has long been
considered the gold standard for diagnosis of PE. Newer multi-detector CT scanners are capable of very rapid scanning of the entire chest in a
single breath hold, providing quality images and visualization of peripheral pulmonary arteries. Studies have recently shown that the sensitivity and
specificity of multi-detector spiral CT scanning for the diagnosis of PE is similar to that of pulmonary angiography. Shown here is an image of a CT
scan with arows demonstrating bilateral pulmonary emboli:

Chest X-ray (choice A) has a very low sensitivity and specificity for PE, and is frequently normal in appearance early in the course of PE.

VQ scan (choice B) is a nuclear study that identifies ventilation and perfusion mismatches in order to establish a diagnosis of PE. The results of VQ
scans are classified as high probability for PE, intermediate probability for PE, low probability for PE, or normal. A result of high probability and
normal are useful to rule in or rule out PE, but most scans tend to fall into the intermediate or low probability for PE categories, which necessitate

further evaluation, usually a pulmonary arteriogram in order to make the definitive diagnosis. In this latter case, much more time is spent making the
diagnosis than with multi-detector spiral CT scanning, and risk of mortality to the patient is therefore increased.

Transthoracic echocardiography (choice C) may detect right sided hemodynamic changes that suggest PE, but specificity for a diagnosis of PE is
very low.

Pulmonary angiography (choice E) has long been considered the gold standard for diagnosis of PE, and still has a role if other tests are nondiagnostic and there is still a strong clinical suspicion for PE. However, it is not practical as a first-line examination due to somewhat limited
availability, expertise required to perform the examination, associated expense, and relative level of invasiveness.

A 58 year old homeless black woman with a past medical history of breast cancer 15 years ago presents to a local walk-in clinic with complaints of
a dry cough and dyspnea on exertion. She states that she sometimes has fevers and chills at night and that her symptoms have been going on for
about a month. Review of systems reveals that she has used recreational intravenous drugs in the past, the last time being 10 years ago. She is
sent to radiology for a chest X-ray, which you are interpreting. The PA chest film is shown.
Which of the following conditions would you include in your differential diagnosis based on this chest film?

1)
2)
3)
4)
5)

Lymphangitic metastases
Sarcoidosis
Histoplasmosis
Pneumocycstis carinii pneumonia (PCP)
All of the above

The correct answer is choice E.

This PA chest film shows the classic reticulonodular pattern associated with interstitial lung disease. The pattern is discerned by X-ray due to
thickening of lung interstitium by the disease process (reticular) with either nodular deposits present due to the disease process, or the appearance
of nodules due to visualizing a thickened interstitial linear opacity on end (nodular). In addition on this film, there are air-bronchograms seen in the
left lower lobe, and fullness of the right hilum.

All of the above answers (choice E) are included in the differential diagnosis of diffuse reticulonodular pattern on chest X-ray and would be
applicable to this patient.

Lymphangitic metastases (choice A) in the lungs is most frequently due to primary tumors of the breast, stomach, thyroid, pancreas, larynx, cervix,
and lung. The pathogenesis is due to the invasion of the lymphatics by hematogenous metastases at the lung periphery. Distribution is usually
uniform throughout both lungs, but may be better seen in the lower lung zones due to the greater volume of lung tissue there. The patient
presented in this case has a past medical history of breast cancer, so this diagnosis is to be included in the differential.

Sarcoidosis (choice B) involving the lung can produce a reticulonodular pattern. It generally is characterized by hilar and mediastinal
lymphadenopathy, which tends to regress as the interstitial lung disease pattern develops. Sarcoidosis is most frequently seen in middle-aged
black females, so would be a distinct possibility in this case.

Histoplasmosis (choice C) is a fungal infection that can also cause a diffuse reticulonodular pattern. Other pathogens in this category include
coccidioidomycosis, cryptococcus, and blastomycosis. A homeless patient is certainly prone to fungal infections.

Pneumocycstis carinii pneumonia (choice D) is an opportunistic infection that afflicts immunocompromised patients, and can present with a diffuse
reticulonodular pattern on chest X-ray. It can also have a more alveolar component and demonstrate a ground-glass appearance in a butterfly
distribution. With this patients past history of IV drug abuse, she could have HIV disease and be immunocompromised, presenting with PCP.

The list of conditions that can produce a reticulonodular pattern on chest X-ray is extensive, but also includes pneumoconiosis silicosis,
asbestosis, talcosis, berylliosis, coal miners disease, etc; connective tissue disorders such as scleroderma, dermatomyositis/polymyositis,

rheumatoid disease, systemic lupus erythematosis, and Sjogrens syndrome; viral pneumonias; pulmonary Langerhans cell histiocytosis;
lymphoma; pulmonary lymphangiomyomatosis; and many others.

A 65 year old woman is referred to a neurologist for progressive complaints of headache, drowsiness, poor cognition, and mild gait disturbance.
Physical examination is remarkable for mild to moderate papilledema bilaterally. The patient is sent to radiology for a CT scan of her brain, an
image of which is shown here. Also of note on lower cuts of the posterior fossa in this CT scan is that the 4th ventricle is normal in size.
What is your diagnosis in this patient?

1) Communicating hydrocephalus
2) Obstructive hydrocephalus

3) Normal pressure hydrocephalus


4) Brain atrophy
5) Intracranial hemorrhage
The correct answer is choice B.

The CT scan shown demonstrates moderate symmetric enlargement of the lateral ventricles with some normal sized and some moderately effaced
sulci peripherally. These findings, in conjunction with the fact that the 4th ventricle is normal in size, confirms a diagnosis of obstructive
hydrocephalus (choice B) with the obstruction lying somewhere in between the lateral and 4th ventricles. The dilated ventricles which are under
pressure exert mass effect on the cerebral hemispheres causing effacement of the sulci. Mass lesions of the midbrain exerting mass effect on the
aqueduct of Sylvius are common causes of obstructive hydrocephalus.

Cerebrospinal fluid (CSF) is produced by the choroid plexes of the lateral and 3rd ventricles and fills the ventricular system, flowing out the
foramina of Lushka and Magendie to cover the cerebral hemispheres in the subarachnoid space, to finally be absorbed by the arachnoid
granulations of the superior sagittal sinus. Overproduction of CSF, under-absorption of CSF, or obstruction of flow of CSF can all cause
hydrocephalus.

Communicating hydrocephalus (choice A) occurs when there is either an overproduction of CSF, such as in the case of a choroid plexus papilloma,
or when there is under-absorption of CSF by the arachnoid granulations. In communicating hydrocephalus, the entire ventricular system is dilated
proportionally. Hence, this choice is an incorrect answer.

Normal pressure hydrochephalus NPH (choice C) is a condition which shows enlargement of all ventricles and normal sized sulci, therefore this
choice is an incorrect answer. There is no elevation of CSF pressure on spinal tap with NPH. The classic triad of dementia, gait disturbance, and
incontinence is associated with this condition. Although more common in the elderly, NPH can also affect younger patients.

In brain atrophy (choice D), all ventricles are enlarged, but the sulci are also enlarged as well. This is due to the global loss of brain tissue. Hence
this choice is also an incorrect answer.

Intracranial hemorrhage (choice E) is an incorrect answer in this case as there is no evidence on this CT scan for hemorrhage which would
manifest as high density material within the calvarium.

A 19 year old male arrives in the radiology department via the emergency room for X-rays of his right forearm to rule out fracture. There is indeed a
nondisplaced, mildly angulated fracture of the mid right ulna. You also notice on the films that the bones are quite sclerotic. You pull up old films on
the patient and see that he has been in two other times over the last 4 years with other fractures. You find a chest film from a year previously and
when viewing it see the classic bone-within-bone appearance of the thoracic spine.
Which of the following disorders that affect bone is included in the differential diagnosis of bone-within-bone?
1)
2)
3)
4)
5)

Osteopetrosis
Transverse growth lines
Sickle cell anemia
Pagets disease
All of the above

The correct answer is choice E.

Osteopetrosis (choice A) is characterized by generalized osteosclerosis, frequent fractures, and the classicbone-within-bone appearance.
Categories of this disease include autosomal recessive infantile osteopetrosis which is lethal, and autosomal dominant adult osteopetrosis.

Transverse growth lines in vertebral bodies (choice B) manifest as a bone-within-bone appearance and may be due to chronic disease in
childhood, malnutrition, or chemotherapy.

Sickle cell anemia (choice C) characteristically shows generalized osteopenia with cortical thinning, increase in bony trabeculae, fish vertebrae
and H vertebrae. Bone-within-bone appearance can also occur and sickle cell anemia therefore is included in the differential.

Pagets disease (choice D) is a sclerotic bone disease that frequently involves the vertebral column causing enlargement of vertebral bodies,

coarsened trabeculae, ivory vertebrae, and picture-frame vertebrae. Bone-within-bone appearance can also occur in Pagets disease and is
therefore included in the differential.

A 69 year old male presents to his physicians office for a routine visit, after not having been to the doctor for 5 years. He does not have any
serious complaints, but just feels a little more fatiqued lately than usual and has lost a couple of pounds. Review of systems is otherwise
unremarkable. Physical examination is unremarkable as well, but hemmocult test is positive for blood in the stool, and a CBC reveals a hematocrit
of 30. The physician sends the patient to radiology for an air-contrast barium enema as part of his initial work-up. You are performing the study, and
obtain the following spot film of the hepatic flexure.
What is the most likely diagnosis in this patient?

1) Colonic muscle spasm


2) Diverticulitis
3) Ischemic stricture

4) Colon cancer
5) Crohns colitis
The correct answer is choice D.

The double-contrast barium enema study spot film show a classic apple core mass lesion of the colon with significant narrowing of the colonic
lumen and ulcerated mucosa, overhanging fairly smooth margins of the mass, and normal colonic mucosa immediately adjacent to the mass. There
is also double tracking of barium seen within the mass superiorly. These findings are most consistent with colon cancer (choice D).

Colorectal cancer is the second leading cause of cancer-related death in the United States. It occurs in 150,000 patients per year in the U. S. and is
responsible for 56,000 deaths per year. Most colon cancers develop from pre-existing adenomatous polyps that undergo malignant transformation;
therefore, screening for colonic polyps has proven very effective in reducing the incidence of colon cancer. The risk of developing colon cancer
from an adenomatous polyp is directly proportional to the size of the polyp. 10% of polyps greater than 1 cm in size are found to be malignant.
Because 1/3 of adenomatous polyps are located proximal to the splenic flexure, it is imperative that the entire colon be screened.
Gastroenterologists and radiologists may disagree on the best method for screening (colonoscopy vs. air-contrast barium enema or virtual
colonoscopy via CT scan), and there are multiple studies which tout the benefits of each exam, but the point is that however it is done, the entire
colon should be screened in people 50 years old and older every 10 years.

Radiologic findings of colon carcinoma on air-contrast barium enema are varied. Tumors may be polypoid, flat, annular, or scirrhous. Polypoid
lesions vary from small to large, pedunculated or not, smooth to lobulated, and may possibly be ulcerated. The larger the lesion, or if ulcerated, the
more likely that it is malignant. Flat lesions are uncommon, and are seen as a uniltateral broad based contour defect, with possible ulceration. Flat
lesions are highly suspicious of malignancy. Annular lesions (apple core) result from circumferential tumors that constrict the colonic lumen. This
type of lesions demonstrates overhanging margins or shoulders of the tumor at the edges and ulceration of mucosa within the narrowed lumen. An
apple core lesion is very highly suggestive of malignancy and should be considered so unless proven otherwise. A saddle lesion is similar, and
basically composed of half an apple core lesion. Scirrhous carcinoma of the colon is a rare form of annular cancer in which there is intense
desmoplastic reaction to the cancer in the bowel wall producing a long segment of mass and luminal narrowing with more gradual margins of the
tumor not demonstrating overhang or shoulder formation.

The differential diagnosis of double tracking of barium includes diverticulitis, Crohns disease, and colon cancer. However, diverticulitis (choice B)
and Crohns colitis (choice E) do not classically demonstrate an annular lesion with overhanging margins and adjacent normal mucosa.

Likewise, colonic muscle spasm (choice A) would not demonstrate overhanging margins, rather the margins of the narrowing would be smooth, the
spasm transient in nature, and the mucosa would be intact.

Ischemic stricture (choice C) due to fibrosis can produce an annular type appearance, but again, with more smooth than overhanging or shelf-like
margins, and likely asymmetry of the lesion with flattening along the mesenteric border and pleating of the antimesenteric border. The mucosa in
this case would also be intact, hence this choice is also incorrect.

A 5 month old infant girl is brought into the emergency room by her mother who states that the baby has had mild symptoms of an upper respiratory
tract infection over the past few days, but that now the baby has suddenly developed a high fever, mild stridor, lethargy, and will not feed. The
infant is sent to the radiology department for X-rays of her neck and you are reading the following lateral neck film.
Which of the following causes of upper airway obstruction in infants is the most likely diagnosis in this case?

1)
2)
3)
4)
5)

Croup
Epiglottitis
Thyroglossal duct cyst
Retropharyngeal abscess
Vascular ring

The correct answer is choice D.

Retropharyngeal abscess (choice D) may occur in infants as a complication of a nasopharyngeal infection, frequently caused by staphylococcus.
This condition may also occur in older children as a result of a penetrating injury to the posterior pharynx. This patients lateral neck X-ray shows
marked widening of the retropharyngeal space with anterior displacement of the airway, and reversal of the normal cervical lordosis, all classic

radiographic signs of a retropharyngeal abscess. In some instances, gas may be seen on X-ray in the soft tissues of the abscess. CT scan may
also be useful in imaging of retropharyngeal abscess, but subjects the patient to significantly more radiation and would require sedation for
scanning. A CT image of a retropharyngeal abscess is shown here:

Croup (choice A) is a laryngotracheobronchitis caused by a respiratory syncitial virus (RSV) infection that also presents with respiratory symptoms,
stridor, and fever. However, the classic radiographic appearance of croup is edema of the subglottic trachea producing the steeple sign or
inverted V appearance of the upper airway, best seen on AP films of the neck. The retropharygeal space is not widened in croup as it is in the
case presented.

Epiglottitis (choice B) may be caused by Hemophilus influenzae, streptococcus A, or pneumococcus. It is a potentially lethal condition, usually in

seen in children greater than 3 years of age presenting with abrupt onset of respiratory distress, stridor and dysphagia. Radiographic findings
demonstrate enlargement of the epiglottis and thickening of the aryepiglottic folds on lateral neck films. The retropharygeal space is also not
widened in epiglottitis as in the case presented.

Thyroglossal duct cyst (choice C) is a congenital anomaly that may present with fever and stridor if the cyst becomes infected and is large, but the
location of the soft tissue swelling on lateral X-ray of the neck would be anterior to the airway somewhere between the hyoid bone and the base of
the tongue.

Vascular rings (choice E) in the neck may present with stridor, but would not necessarily be associated with symptoms of a respiratory infection,
and would not demonstrate such significant soft tissue swelling of the retropharyngeal space.

A 37 year old woman with a very strong positive family history for breast cancer comes into the radiology department for her first mammogram. The
mammogram shows extremely dense breast tissue which is bilaterally symmetric, with no definable abnormality. The patients physician is
concerned enough that he asks you to perform an adjuvant breast MRI as part of her initial breast evaluation.
In which of the following scenarios is breast MRI also useful?
1)
2)
3)
4)
5)

Screening of the contralateral breast which is normal on mammogram in a patient with a newly diagnosed breast cancer
For improved evaluation of breast tissue in women with breast implants
As an aid in determining extent and multifocality of DCIS
In breast cancer patients who are postlumpectomy with positive or close margins
All of the above

The correct answer is choice E.

Breast MRI is useful in and supported by the American College of Radiology for all of the scenarios listed above (choices A D). Other indications
include preoperative evaluation of suspected cancers close to the chest wall to assess invasion, as an adjuvant for monitoring the response to
chemotherapy, suspicion of recurrence of breast cancer when mammographic and sonographic findings are inconclusive, in cases of metastatic
cancer suspected to be of breast origin when mammographic and sonographic findings are inconclusive, and in the case of suspected cancer in
patients with postoperative tissue reconstruction.

A 52 year old marathon runner comes into orthopedic clinic complaining of progressive right knee pain. The orthopedist examines him and
suspects that the patient has osteoarthritis. X-rays of his right knee are obtained which you are interpreting.
Which of the following radiographic findings would you expect to see on this patients knee films?
1)
2)
3)
4)
5)

Subchondral sclerosis
Medial joint space narrowing
Subchondral cysts
Osteophyte formation
All of the above

The correct answer is choice E.

Osteoarthritis, also referred to as degenerative arthritis, most commonly affects the knee joint. Its pathogenesis is due to repeated stress and
alterations in normal biomechanical function of the joint. Typical radiographic signs of osteoarthritis in the knee are:

1.) Subchondral sclerosis (choice A) or increased bone density adjacent to articular cartilage, appearing early in the medial tibial plateau;
2.) Joint space narrowing (choice B) due to loss of articular cartilage, which is more common in the medial portion of the knee joint and may result
in a varus deformity if severe;
3.) Subchondral cysts (choice C) which are characteristically small in osteoarthritis, the presence of large subchondral cysts would suggest an
alternative diagnosis of rheumatoid arthritis or calcium pyrophosphate dihydrate disease (CPPD) also referred to as pseudogout;
4.) Osteophyte formation (choice D) are protrusions of bone at the articular margins of a joint which form as a result of the reparative attempt of
bone and cartilage to preserve the joint space.

An 11 year old boy goes in to see his pediatrician for his yearly visit. His mother states that the boy seems to fatigue more readily than his siblings
and sometimes complains of leg cramps. On physical examination the pediatrician notes a soft, continuous systolic murmur in the upper back, and
decreased pedal pulses. ECG shows mild left ventricular hypertrophy. His imaging work-up includes the attached film.
Which of the following is the most likely diagnosis in this case?

1)
2)
3)
4)
5)

Aortic stenosis
Aortic aneurysm
Patent ductus arteriosus (PDA)
Transection of the aorta
Coarctation of the aorta

The correct answer is choice E.

Coarctation of the aorta (choice E) likely has a genetic component, and depending upon its severity, can present in the neonate, infant, child,
adolescent or adult. Neonates and infants tend to present with congestive heart failure. It is a condition associated with a bicuspid aortic valve in up

to 50% of cases. This case demonstrates a short, discrete shelf-like narrowing of the descending aorta in the region of the ligamentum arteriosum.
The more severe form which presents in infancy would have more of a longer, more tubular like narrowing on imaging. Echocardiography and MRI
are also very useful in the diagnosis of this condition.

Transection of the aorta (choice D) due to trauma by a shearing injury tends to occur at the level of the ligamentum arteriousum which is a fixation
point for the descending aorta. Imaging would reveal intimal tear, false aneurysm, and possible aortic dissection.

Aortic stenosis (choice A) refers to stenosis of the aortic valve and not the aorta itself.

There is no evidence of a saccular or fusiform aortic aneurysm (choice B) on this image, nor is there evidence of a patent ductus arteriosus (choice
C).

You might also like